0% found this document useful (0 votes)
123 views63 pages

Pbl3b End of Year Solved

Uploaded by

medaaaf1
Copyright
© © All Rights Reserved
We take content rights seriously. If you suspect this is your content, claim it here.
Available Formats
Download as PDF, TXT or read online on Scribd
0% found this document useful (0 votes)
123 views63 pages

Pbl3b End of Year Solved

Uploaded by

medaaaf1
Copyright
© © All Rights Reserved
We take content rights seriously. If you suspect this is your content, claim it here.
Available Formats
Download as PDF, TXT or read online on Scribd
You are on page 1/ 63

ACADEMIC COMMITTEE PBL3B…2025

BLOCK 3.1 FEVERS, INFECTIONS, c. Streptococcus pyogenes from


AND INFLAMMATIONS I Staphylococcus aureus
1. The following are true of Micrococci d. Streptococcus pyogenes from
except? Streptococcus faecalis
a. Gram positive anaerobes
b. Opportunistic pathogen 6. Which one of the following is considered
c. Skin commensals a virulence factor for Staphylococcus
d. They are catalase positive aureus?
a. A heat-labile toxin that inhibits
2. Your patient is a 70 years-old man who glycine release at the internuncial
recently had surgery suddenly develops neuron
urinary tract infection. Which of the b. A heat-stable enterotoxin
following organism is MOST likely to be c. Protein A that binds to the Fc
the cause? portion of IgG
a. Actinomyces Israeli d. Resistance to novobiocin
b. Enterococcus faecalis
c. Proteus spp 7. Your patient had a subacute bacterial
d. Pseudomonas auroginosa endocarditis caused by member of the
Viridans group of streptococci. Which
one of the following sites is MOST likely
3. Six hours after a delicious thanksgiving to be the source of the organism?
dinner of light soup and potatoes, meat a. Colon
sauce and egg pie topped with ice b. Oropharynx
cream, the Amidu family of four c. Skin
experienced vomiting and diarrhoea. d. Urethra
Which one of the following organisms
is MOST likely to cause these 8. The MOST Important contribution of the
symptoms? ccapsule of Streptococcus pneumoniae
a. Campylobacter jejuni virulence is to
b. Salmonella enientidis a. accelerate tissue invasion by its
c. Shigella flexneri collagenase like activity
d. Staphylococcus aureus b. inhibit polymorphonuclear
leukocytes
4. The occurrence of scarlet fever is due to c. prevent dehydration of the
a. Coagulase organisms on mucosal surfaces
b. Erythrogenic toxin d. retard phagocytosis by
c. Pyogenic toxin polymorphonuclear leukocytes
d. Streptolysin
9. Acute glomerulonephritis is a
5. The coagulase test, in which the bacteria nonsuppurative complication that follows
cause plasma to clot, is used to infection by which one of the following
distinguish? organisms?
a. Staphylococcus aureus from a. Streptococcus agalactiae
Staphylococcus epidermidis b. Streptococcus faecalis
b. Staphylococcus epidermidis from c. Streptococcus pneumoniae
Neisseria meningitidis d. Streptococcus pyogenes
1
ACADEMIC COMMITTEE PBL3B…2025

10. Which one of the following is NOT an b. Grain positive Cocci、


important characteristic of the Lancefield c. Common contaminant of blood
group B streptococci? cultures
a. Beta-hemolytic d. Spore forming organism
b. Cause pyoderma (impetigo)
c. Cause neonatal sepsis and 16. Erysipeloid is caused by
meningitis a. Erysipelothrix rhusiopathiae
d. Vaginal carriage is 5-25% in b. Streptococcus agalactiae
normal women of childbearing c. Streptococcus mitis
age d. Streptococcus pyogenes

11. Which one of the following is NOT an 17. The following statements are correct
important feature in pulmonary EXCEPT
anthracis? a. B- haemolysis is the partial lysis
a. Haemoptysis of red blood cells due to
b. High fever bacterial hemolysins
c. Lymphadenopathy b. Scalded skin syndrome is the
d. Pharyngitis same disease scalatina
c. Susceptibility to novobiocin is
12. In which of the following infections will used to differentiate between
death occur in few hours with 80% CoNS
mortality rate in treated cases? d. Viridans streptococci commonly
a. Listeriosis cause subacute endocarditis
b. Pulmonary anthracis
c. Rocky Mountain spotted fever 18. Which of these organisms causes
d. Tularemia neuropathy in neonates?
a. Erysipelothrix rhusopathiae
13. Of the organisms listed below, which one b. Listeria monocytogenes
is the MOST frequent bacterial cause of c. Micrococcus agalactiae
pharyngitis? d. Streptococcus viridans
a. Neisseria meningitidis
b. Staphylococcus aureus 19. The most currently favoured
c. Streptococcus pneumoniae theory/mechanism of Streptococcal
d. Streptococcus pyogenes sequelae is
a. Autoimmune response to
14. Quellung reaction is a serologic test various streptococcal antigens
definite of b. Production of Streptokinase
a. Meningicoccus c. Tissue injury in both diseases is
b. Micococcus tied directly to Streptococcal
c. Pneumococcus invasion of tissues
d. Staphylococcus d. The action of toxins SLO and SLS

15. The following statements are accuratee 20. Considering the principles of viral
about Bacillus cereus EXCEPT nucleic acid, which of these is correct?
a. Associated with "Fried Rice a. Purified negative sense DNA
Syndrome" genome is infective
2
ACADEMIC COMMITTEE PBL3B…2025

b. Purified negative sense RNA ganglion?


genome is infective a. Cytomegalovirus ๖
c. Purified positive sense DNA b. Epstein Barr virus
genome is infective c. Herpes simplex type 2
d. Purified positive sense RNA d. Varicella Zoster
genome is infective
26. When latent varicella zoster virus is
21. An electron micrograph prepared from a reactivated, the individual will be
sample taken from the oropharynx of a diagnosed with
patient showed protruding fibres at a. Chicken pox
penton vertex capsomere of a naked b. Encephalitis
icosahedral viral particle. Which of these c. Pneumonitis
viruses will you like consider it to be? d. Shingles
a. Adenovirus
b. Parvovirus 27. The hepatitis B viral genome is unique
c. Polio virus since it has a
d. Rhinovirus a. Double stranded RNA genome
b. Fully double stranded RNA
22. The unique and most distinguishing genome that can be used as a
feature of the parvovirus is that vector
a. It has a double helix RNA virus c. Partially double stranded DNA
b. It has a hairpin RNA genome genome
c. It has single stranded DNA d. Partially double stranded RNA
genome genome
d. It’s helical nucleocapsid is 5.6
kilo base pairs 28. Which of these contagions have mild
effect on the human being?
23. A person presenting with ulcer and a. Molluscum contagiosum
blisters on genitalia with painful urination b. Monkey pox
may be suffering from an infection with c. Vaccinia
which of the following viral particles? d. Variola
a. Cytomegalovirus
b. Herpes simplex type 1 29. The pox viruses may be unique because
c. Herpes simplex type 2 they are
d. Herpes simplex types 1 and type 2 a. DNA viruses that bud through the
cell membrane
24. Which of these viruses usually cause b. DNA virses that bud through the
most of the neonatal encephalitis cases? cell endoplasmic reticulum
a. Epstein Barr virus c. DNA viruses that replicate fully
b. Herpes simplex type 1 in the cytoplasm
c. Herpes simplex type 2 d. DNA viruses that replicate in the
d. Herpes simplex types 1 and 2 nucleus

25. The Herpesviridae family is well noted 30. A general but remote risk of the Sabin
for latency, but which virus of the group vaccine of polio is that
become latent in the sensory nerve a. The attenuated viruses can
3
ACADEMIC COMMITTEE PBL3B…2025

revert to wild virulent types d. Cephalothin


b. The strains used could reassort
with other enterovirus genomes 36. The following antibiotics are cell
c. The strains used have reservoirs membrane inhibitors:
close to humans a. Ampicillin
d. They are resistant to formalin and b. Gentamicin
not properly attenuated c. Chloramphenicol.
d. Polymyxin B
31. Which virus may be implicated when
children present with gastroenteritis, 37. Which of the following match of
febrile exanthema, and upper respiratory antimicrobials with their mode of action
tract infection? is NOT correct?
a. Adenoviruses a. Inhibition of DNA
b. Echovirus synthesis…Aminoglycosides
c. Piconavirures b. Inhibition of folic acid
d. Reoviruses pathway…Trimethoprim
c. Inhibition of cell wall
32. A woman infected with rubella in the first synthesis…Cephalosporins
trimester of her pregnancy faces which of d. Interference with cytoplasmic
the following consequences? membrane…Polymixin
a. Cardiac abnormalities
b. Miscarriage of the pregnancy 38. These are bases for the classification of
c. Paralysis antibiotics EXCEPT?
d. Physical and mental retardation a. Chemical structure
b. Mechanism of action
33. Severe attack of one of the following c. Rate of absorption
viruses on male epididymis and lestes can d. Spectrum
lead lo sterility e. Type of activity
a. Hepatitis E
b. Measles 39. A laboratory raised kissing bug is made
c. Mumps to feed on a person suspected with
d. Rubella Chagas disease. The vector is examined
for the development of trypomastigote.
34. When viral particles leave an infected Which form of diagnosis is this?
host cell to infect another cell of a a. molecular diagnosis
different host it is termed b. parasite culture
a. Contamination c. serological diagnosis
b. Release d. xenodiagnosis
c. Shedding
d. Transmission 40. A 38-year-old patient reported with
complications of anaphylactic shocks
35. Identify the 3nl generation cephalosporin from hydatid cyst disease. Ultrasound
among these drugs and MRI scan shows pleural perforation
a. Cefaclor from ruptured hydatid cyst. Which is the
b. Ceftriaxone most likely parasitic agent responsible for
c. Cephalexin the clinical presentations.
4
ACADEMIC COMMITTEE PBL3B…2025

a. Taenia saginata form is most likely to be seen from post-


b. Tinea solium mortem tissue sections?
c. Echinococcus granulosum a. Tachyzoites
d. Ancylostoma duodenale b. Bradyzoites
c. Oocyst
41. A patient was treated for malaria. Few d. Trophozoites
months later he had a relapse without any
history of Plasmodium transmission since 46. Which of the following about fever
his last treatment, which of the following periodicity in Plasmodium infection is
species of Plasmodium could be accurate?
responsible for the relapse? a. Corresponds to the time from
a. vivax and ovale RBC invasion by merozoites to
b. falciparum and vivax schizont rupture
c. nalaria and falciparum b. Associated with movement of
d. ovale and falciparum merozoites from the liver into (he
periphery
42. Sirigu is a village situated close to a lake. c. It is the same for all plasmodium
The inhabitants are noted for open species
defecation and urinating inside the water d. Entry of sporozoites and invasion
when swimming. Which of the following of the liver
parasitic infections would be endemic in
Sirigu? 47. Schistosoma haematobium infection is
a. Elephantiasis characterised by the following disease
b. River blindness manifestations EXCEPT
c. Schistosomiasis a. Damages the wall of mesenteric
d. Trypanosomiasis venules
b. Induces cancer in the bladder
43. Helminth infection are generally c. Hematuria
associated with increase in the... d. Obstructive uropathy
a. Basophils
b. Eosinophils 48. Regarding cysticercosis, which of the
c. Neutrophils following is accurate?
d. B lymphocytes a. Transmission is through ingestion
of pork contaminated with adult
44. An outbreak of mild intestinal distress, Taenia solium
sleeplessness, perianal itching, and b. Acquired by ingesting food
anxiety has broken out among preschool contaminated with eggs of Taenia
children in a private home. The most saginata
likely cause of this condition is... c. Acquired through ingestion of
a. Trichomonas vaginalis food contaminated with Taenia
b. Enterobius vermicularis solium eggs
c. Ascaris lumbricoides d. Eating beef contaminated with
d. Necatar americanus Taenia saginata.

45. Regarding the diagnosis of 49. Early during T. Brucei gambiense


Toxoplasmosis, which morphological infection, replication of the parasite in the
5
ACADEMIC COMMITTEE PBL3B…2025

CSF results in dysregulated rashes


inflammation. Which of the following c. lnfestation of skin by the skin
products from the dysregulatcd burrowing into Sarcoptes scabei
inflammation induces the ‘sleeping d. Transmission is by direct contact
sickness’ syndrome? with an infected person
a. Prostaglandin D2
b. Interleukin-l0 54. Trichomonas vaginalis is transmitted
c. Tissue growth factor beta (TGF-P) primarily by direct sexual contact but not
d. Interferon gamma through orofaecal routes. Which of the
following morphological adaptations
50. The endosymbiotic association between support this phenomenon?
Onchocerca volvulus and a bacterium has a. The presence of cyst and
been reported to increase the trophozoites stages
pathogenicity of the parasite to cause b. The possession of only
blindness. What is the name of the trophozoite stage
bacteria? c. Destruction of the epithelial
a. Wolbachia barriers
b. Cyanobacteria d. Possession of flagella for
c. Staphylococcus aureus movement
d. Escherichia coli
55. Mycoplasma pneumoniae is an important
51. The profound swelling of the scrotum or cause of atypical pneumonia. Regarding
legs called 'elephantiasis’ is caused by: this organism, which one of the following
a. Onchocerca volvulus is the most accurate?
b. Ascaris lumbricoides a. Amoxicillin is the drug of choice
c. Necator americanus for pneumonia caused by this
d. Wuchereria bancrofti organism.
b. Antibody in a patient's serum
52. The treatment of Onchocerciasis requires will agglutinate human red
the use of antibiotic and anti-helminthic blood cells at 4°C, but not at
drugs because of its endosymbiont 37°C.
relationship with a bacterium. Which of c. Gram stain of the sputum reveals
the following combination therapies is small gram-negative rods.
recommended to kill both microfilaria d. It is an obligate intracellular
and macrofilaria? parasite that can only grow within
a. Doxycycline and ivermectin human cells in the clinical
b. Albendazole and ivermectin laboratory.
c. Tetracycline and ivermectin
d. Ivermectin and praziquantel 56. Which one of the following is the drug of
choice for atypical pneumonia caused by
53. Which of the following about Scabies is M. pneumoniae?
inaccurate? a. Amoxicillin
a. Parasitic infection of caused by b. Azithromycin
Sarcoptes scabei that burrows in c. Ceftriaxone
the skin d. Gentamicin
b. Produced itches and purple-like
6
ACADEMIC COMMITTEE PBL3B…2025

57. Your patient is a 20-year-old man with a 60. Regarding Q-fever, which one of the
urethral discharge. Gram stain of the pus following is most accurate?
reveals many neutrophils but no bacteria. a. The causative organism is
You suspect this infection may be caused transmitted by tick bite.
by Chlamydia trachomatis. Which one of b. The natural habitat of the
the following is the laboratory result that causative agent is the white-
best supports your clinical diagnosis? footed mouse.
a. Fluorescent-antibody staining c. The diagnosis is made primarily
of cytoplasmic inclusions in by Gram stain and culture on
epithelial cells in the exudate chocolate agar.
b. Gram stain of the pus reveals d. Occupations that predispose
small gram positive rods. people to Q fever include
c. The organism produces beta- veterinarians and abattoir
hemolytic colonies on blood agar workers
plates when incubated
aerobically. 61. The following bacteria are obligate
d. The organism produces alpha- intracellular parasites, except?
hemolytic colonies on blood agar a. Chlamydia trachomatis
plates when incubated b. Chlamydophila psittaci
anaerobically, c. Haemophilus influenzae
d. Mycoplasma pneumoniae
58. Regarding Chlamydia, which one of the
following is the most accurate? 62. Non-gonococcal urethritis may be caused
a. Lifelong immunity usually by
follows an episode of disease a. Chlamydia trachomatis
caused by these organisms. b. Legionella pneumophila
b. The reservoir host for the three c. Mycoplasma hominis
species of Chlamydia that cause d. Ureasplasma urealyticum*
human infection is humans.
c. Their life cycle consists of 63. The most common cause of atypical
elementary bodies outside of pneumonia is
cells and reticulate bodies a. Chlamydia trachomatis
within cells, b. Chlamydophilia psitacci
d. They can only replicate within c. Coxiella burnetti
cells because they lack the d. Ricketssia rickettsia
ribosomes to synthesize their
proteins. 64. Neonatal conjunctivitis is not associated
with
59. Which one of the following is the drug of a. Neisseria gonorrhoeae
choice for sexually transmitted disease b. Chlamydia trachomatis
(urethritis, cervicitis} caused by c. Mycoplasma hominis
Chlamydia trachomatis? d. Chlamydophilia psitacci
a. Ampicillin
b. Azithromycin 65. Newborn associated Chlamydiosis does
c. Ciprofloxacin not include
d. Metronidazole a. Blindness
7
ACADEMIC COMMITTEE PBL3B…2025

b. Conjunctivitis c. Mycotoxicosis
c. Pneumonitis d. Food
d. Reiter’s syndrome
70. Which of the following genera of a fungi
66. The following diseases are arthropod is not a member of the dermatophytes?
borne except.... a. Malassezia
a. Q- fever b. Epidermophyton
b. Rickettsiasis c. Trichophyton
c. Rocky mountain spotted fever d. Microsporum
d. Typhus
71. The main reason why some bacteria are
67. Which of the following is true about anaerobes (ie they cannot grow in the
Chlamvdia physiology? presence of oxygen) is because
a. Elemental- body is metabolically a. they do not have sufficient
active catalase and superoxide
b. Elementary bodies undergo active dismutase,
replication b. they have too much ferrous ion
c. Reticulate body is the infectious that is oxidized to ferric ion m the
form presence of oxygen.
d. Species are identified by major c. they have unusual mitochondria
outer membrane proteins that cannot function in the
presence of oxygen.
68. Which of the following statements about d. transcription of the gene for the
the medical importance of the fungus cell pilus protein is repressed in the
wall is accurate? presence of oxygen
a. Fungi are sensitive to drugs such
as penicillin and cephalosporin 72. Which one of the following sets consists
because their cell wall is of bacteria that are both anaerobes?
composed mainly of chitin. a. Actinomyces israeli and Serratia
b. The fungal cell wall is also marcescens
composed of sterols, which is the b. Campylobacter jejuni and Vibrio
target site for drugs such as cholerae
anphotericin B. c. Clostridium perfringens and
c. The fungal cell wall is composed Batcteroides fragilis
of N- acetyl glucosamine and B- d. Mycobacterium tuberculosis and
glucan, and Caspofungin Pseudomonas aeruginosa
targets B-glucan.
d. The synthesis of the fungal cell 73. Which of the statements about fungi is
membrane is inhibited by azole inaccurate?
drugs as the cell membrane is a. The natural habitat of the all
composed of cholesterol. fungi is the environment.
b. All fûıngi are eukaryotic in
69. Which of the following is not one of the nature.
medical importance of fungi? c. Fungi can be cultured on
a. Mycosis Saboraud's medium because
b. Allergies Saboraud’s medium is a selective
8
ACADEMIC COMMITTEE PBL3B…2025

medium for fungi in the yeast form in the


d. Serological tests are mostly environment.
available for diagnosing systemic c. They occur through the
mycosis inhalation of asexual spores
into the lungs.
74. Which of the following statements about d. They are mostly
fungal pathogenesis is most accurate? asymptomatic even though
a. Ingestion of Amanita mushrooms dissemination may occur
causes hepatic carcinoma by leading to death.
inhibiting mRNA synthesis.
b. Ingestion of food coııtaıninated 77. Which of the following systemic mycosis
with fungi, Mycetismus, can result is known as the South American
in mutation of p53 gene leading to blastomycosis?
cancers. a. Paracoccidioidomycosis
c. Allergies are characterized by b. Blastomycosis
eosinophilia and bronchodilation c. Histoplasmosis
mediated by IgE d. Coccidioidomycosis
d. Immune response to mycosis
leads to the formation of 78. Regarding coccidioidomycosis and C.
granulomas sometimes with the immitis, which one of the following is
production of pus. most accurate?
a. C. immitis is a mold in the soil and
75. Which of tbc following statements about a yeast in the body.
mycetoma is most appropriate? b. Dissemination mostly occurs in
a. Mycetoma caused by fungi exists pregnant women in their third
as either eumycetoma or trimester,
actinomycetoma. c. Mycosis could be prevented by
b. Mycetoma is caused by normal not travelling to African countries
flora such as Petriellidium and d. The nodules of erythema
Madurella nodosum are a typical finding in
c. Mycetoma is characterised by disseminated
the suppuration of pus coccidioidomycosis.
containing coloured granules
through the sinuses. 79. Regarding histoplasmosis and H.
d. Antifungals such as capsulatum, which one of the following
Amphotericin B and Itraconazole is most accurate?
are drug of choice. Surgery is not a. in bone marrow aspirates, H
required. capsulatum is found as a yeast
within macrophages.
76. Which of the following statements about b. The laboratory diagnosis is made
systemic mycosis is inaccurate? by seeing micronidia when
a. They are non-communicable incubated at 37C
endemic infections. c. Histoplasmosis occurs primarily
b. The causative fungi mostly in the tropical areas of Latin and
exist in dimorphic forms; South America.
mould form in the body and d. Most infections are acquired by
9
ACADEMIC COMMITTEE PBL3B…2025

ingesting food accidentally inaccurate?


contaminated with fungal spores a. Transmission is from soil,
from the soil. contaminated meat and
unsterilized canned foods
80. Regarding C. albicans, which one of the b. Ascending weakness and
following is inaccurate? paralysis usually occur.
a. Insufficient neutrophils c. Treated with a heptavalent
predispose individuals to antitoxin with respiratory-
disseminated Candida infections. support
b. It exists as a yeast on mucosal d. Can be prevented by consuming
surfaces but forms pseudohyphae properly cooked fool
when it invades tissue.
c. C. albicans are exclusively 85. Which of the following gram-negative
found in the soil bacteria can not reduce nitrates to
d. C. albicans causes chronic nitrites?
mucocutaneous candidiasis a. Escherichia coli
during defective Th-17 immune b. Klebsiella pneumoniae
response. c. Proteus vulgaris
d. Salmonella paratyphi
81. Immune reconstitution inflammatory e. Yersinia pestis
syndrome i.e severe condition that occurs
when treating AIDS patients with 86. Which of the gram-negative bacteria
HAART in the presence of would diffuse from the point of
a. Cryptococcus meningitis inoculation?
b. Candidiasis a. Escherichia coli
c. Aspergillosis b. Klebsiella
d. Zygomycosis c. Pseudomonas aeruginosa
d. Salmonella paratyphi
82. Which of the following is not a sign of
anaerobic infections during physical 87. Which of the following medium is used
examinations? to isolate Salmonella species and
a. Foul smell Klebsiella species?
b. Necrotizing tissues a. Blood agar
c. Erythema nodosum b. Chocolate agar
d. Gas in tissue c. Citrate agar
d. MacConkey agar
83. Which one of the following bacteria
causes disease characterised by a 88. Most enteric bacteria possess antigen that
pseudomembrane? can be used to identify them into groups.
a. Listeria monocytogenes The O antigen is...
b. Clostridium perfringens a. Capsular, heat stable located in
c. Bacillus cereus the cell wall
d. Clostridium difficile b. Capsular, heat labile located in
the cell wall
84. Which of the following statements c. Somatic, heat labile located in the
regarding Botulism and C. botulinum is wall
10
ACADEMIC COMMITTEE PBL3B…2025

d. Somatic heat stable located in b. K. pneumoniae


the cell wall c. K. rhinosclermatis
e. d. K. sinatis
89. Which of the following E. coli serotypes
is responsible for gastrointestinal 94. Which antigenic structure is involved in
infection in children characterized by the bonding together of gonococci within
mucus in stool? colonies?
a. Enteroaggregative E. coli a. Lipo-oligosaccharide
b. Enterohaemorrhagic E. coli b. Opa proteins
c. Enteroinvasive E. coli c. Pili
d. Enteropathogenic E. coli d. Por proteins
e. Enterotoxigenic E. coli
95. Toxicity in gonococcal infections is
90. Travelers diarrhoea is associated with largely due to the endotoxic effects of
food contaminated by lipo-oligosaccharide. Which of the
a. Enteroaggregative E. coli following is accurate?
b. Enterohaemorrhagic E. coli a. N. gonorrhoeae that lacks
c. Enteroinvasive E. coli serum resistance are able to
d. Enteropathogenic E. coli spread
e. Enterotoxigenic E. coli b. N. gonorrhoeae that possesses
serum resistance remain localized
91. Which of the following gram-negative in the genital tract
bacteria has the respiratory tract as a c. N. gonorrhoeae generates serum
major portal of entry and is isolated from resistance by modifying lipo-
the respiratory tract of up to 80% of post- oligosaccharides on their surface.
operative patients developing d. Serum resistance interfere with
bacteremia? proper formation of the capsule
a. Serratia marcescens attack complex during the
b. Serratia liquefaciens complement cascade.
c. Serratia adorifera e. Scrum resistance may impact
d. Shigella marcescens intracellular killing of gonococci
within neutrophils
92. Yersinia are bacteria that are oxidase-
negative, catalase positive, whose cells 96. Pili are made up of stacked pilin proteins
are primarily gram-negative straight which aid in the attachment to host cell
rods. Which of the following can infect and resistance to phagocytosis. Which
rodent and man? part is most prominent in the immune
a. Yersinia ofensis response in gonococcal infection?
b. Yersinia enterocolitica a. The amino terminal of the pilin
c. Yersinia pestis molecule
d. Yersinia pseudotuberculosis b. The amino acid sequence near the
mid portion
93. Which of the following Klebsiella species c. The amino acid sequence near
is mostly associated with nosocomial the carboxyl terminal
infections? d. The hydrophobic part of the
a. K. oxyloca amino acid sequence
11
ACADEMIC COMMITTEE PBL3B…2025

e. All the parts of the amino acid d. Enrichment medium will select
sequence for the growth of gram-positive
cocci
97. Por proteins extends through the e. It will enhance the identification
gonococcal cell membrane. Which of the of haemolytic bacteria.
following is not a function of por
proteins? 100. Which of the following Moraxella
a. Expression of por proteins make species cause sinusitis?
different strains antigenically a. M. Catarrhalis
different b. M. Faciens
b. It occurs in trimers to form pores c. M. Lacunata
on the surface through which d. M. Non liquifaciens
some nutrients enter the cell
c. Por protein on the gonococcal
surface is the same surface BLOCK 3.2 CARDIOVASCULAR AND
structures as human cells RESPIRATORY PATHOLOGY
d. Por proteins may impact 1. A 72-year-old man had had colorectal
intracellular killing of gonococci surgery at the Tamale Teaching Hospital for
within neutrophils by preventing sigmoid colon cancer. Which of the under
lysosome fusion listed parameters must be strictly monitored
a. Blood pressure
98. IMViC reactions are a set of four b. Oxygen saturation curved
reactions employed in the identification c. Input output chart
of Enterobacteriaceae. These include d. Pulse rate
Indole test, Methyl Red test, Voges
Proskauer and Citrate utilization test. 2. What is the total intracellular fluid volume
Which of the following organism cannot in a 24-year medical of body weight 58 kg,
be identified using these tests? on admission at the Surgical unit in the
a. Escherichia coli Tamale teaching hospital?
b. Klebsiella pneumoniae a. 23L
c. Proteus vulgaris b. 36L
d. Pseudomonas aeruginosa c. 35L
e. Salmonella Typhi d. 20L

99. A specimen taken from a draining 3. Which of the following conditions will
abdominal abscess must be kept in an result in shock without loss of body fluid?
enrichment medium because a. Gram-negative sepsis
a. Enrichment medium contains b. Severe haemorrhage
blood which is nutritious for c. Head injury
fastidious bacteria d. Vomiting in pregnancy
b. Most likely pathogens are
anaerobes and cannot grow in 4. Most pulmonary emboli arise from?
the presence of oxygen a. Deep arteries of the lower extremities;
c. It will enhance the growth of the most fatal ones from ilio-femoral veins,
bacteria in the abscess b. Pelvic venous plexus
c. Right side of the heart.

12
ACADEMIC COMMITTEE PBL3B…2025

d. Deep veins of the lower extremities; b. Speech deficit


most fatal ones form in ilio-femoral veins c. Memory loss
d. Subarachnoid haemorrhage
5. Which of the following types of shock is
associated with dry and warm extremities? 11. Mitral regurgitation may result in the
a. Haemorrhagic following complications except?
b. Septic shock a. Left ventricular failure
c. Anaphylactic shock b. Dilation of the aortic valve cusps
d. Neurogenic shock c. Right sided heart failure
d. Pulmonary hypertension
6. Which of the following congenital heart
diseases is associated with foetal alcohol 12. Occlusion of a right dominant coronary
syndrome? artery in a patient may result in all of the
a. Ventricular septal defect following except?
b. Patent ductus arteriosus a. Pulmonary thromboembolism
c. Atrial septal defect b. Lateral ventricular infarction
d. Coarctation of the aorta c. Posterior septal infarction
d. Cerebral infarction
7. For a 78-year-old woman with endometrial
cancer on admission at the gynae ward of the 13. Early and continuous cyanosis is
Tamale teaching hospital, the common place characteristic of the following congenital
to check for oedema is? abnormalities except
a. Abdomen A Complete transposition of the great vessels
b. Lungs b. Patent ductus arteriosus
c. Lumbo-sacral region c. Total anomalous venous return
d. Lower limbs d Tetralogy of Fallot

8. Which of the following medical causes of 14. Acute left sided cardiac failure may occur
oedema in a 5-year girl is associated with in the following pathological situations
raised intravascular hydrostatic pressure? except?
a. Congestive heart failure a. Massive pulmonary embolism
b. Liver cirrhosis b. Rheumatic mitral valve stenosis
c. Nephrotic syndrome c. Myocardial rupture following infarction
d: Malnutrition d. Papillary muscle rupture

9 Which of the following types of allergic 15. The following under listed statement
edema is life threatening? concerning the rupture of the myocardium
a. Hive fever following an infarction are true except?
b. Urticarial a. Characteristically occurs 36-48 hours
c. Angioneurotic after infarction
d. Anasarca b. May cause cardiac tamponade
10. A thrombus from the right internal carotid c. May cause ventricular septal defect
artery in a 90-year hypertensive, is most d. Predisposes to arrhythmias
likely to be associated with which of the
following presentation? 16. Which of the following statements about
a. Intentional tremors Fallot's tetralogy is true
13
ACADEMIC COMMITTEE PBL3B…2025

a. Pulmonary stenosis C. Lenticulostriate artery


b. Overriding aorta d. Posterior cerebral artery
c. Left ventricular hypertrophy
d. a and b 23 Severe atherosclerosis involving branches
of the posterior cerebral artery that supply the
17. Regarding infective endocarditis, which occipital lobe will result in?
of the following is a. Expressive aphasia
a. most commonly caused by streptococcus b. Cortical type visual lost
faecalis infection c. Right hemiparesis
b. May cause Osler's nodes in the fingers d. Confusion
c. May cause glomerulonephritis
d. All of the above 24. Which of the following is not a likely
cause of Cerebral oedema?
18. The following are true regarding a. Anoxia
abnormalities of rheumatic heart diseases b. Hyperglycemia
except? c. Hypertensive encephalopathy
a. Mitral valve is the commonest valve d. Abscesses
affected
b. Pericardial involvement is rare 25. The thickness of left ventricular wall in
c. Left atrial endocardial scarring is a left ventricular hypertrophy at least
frequent feature a. 19mm
d. None of the above c. 15mm
c. 17mm
19. Which of the following underlisted d. All of the above
conditions is/are risk factors of recognizable
emboli 26. The features of tetralogy of Fallot areas
a. Infective endocarditis under except
b. Prosthetic valve a. VSD
c. Myocardial infarction b. Displacement of aorta to right to override
d. All of the above the VSD
c. Pulmonary stenosis
20. The following are true with regards to d. Left ventricular hypertrophy
ischemic cerebral infarction except
a. May complicate hypertension 28. Non-infarct effects of myocardial
b May result from Libman sacks ischemia are as under listed except
endocarditis a. Sudden cardiac death
c. May complicate atherosclerosis b. Angina pectoris
d. The earliest gross change appears as a pale, c. Subendocardial infarcts
soft area with loss of the normal demarcation d. Chronic ischaemic heart disease
between gray and white matter
29. Which of the following statements
21. Rupture of which of the following vessels regarding infarcts is least true?
in a hypertensive will lead to intracerebral A. Left ventricle
haemorrhage? b Right ventricle
a. Berry aneurysm c. Left atrium
b. Lacunars artery d. Right atrium
14
ACADEMIC COMMITTEE PBL3B…2025

a. Carditis
30. CKMB2:CKMB1 ratio sensitive for the b. Polyarthritis
diagnosis of acute MI is: C. Raised C-reactive proteins
a. >0.5 d. Subcutaneous nodules
b. >1.0
c. 1.5 37. Vegetations of the following types of
d. >2.0 endocarditis are generally not friable except
that of?
31 Chronic ischaemic heart disease is most A. Rheumatic endocarditis
often due to: b. Libman-Sacks endocarditis
a. Coronary atherosclerosis c. Subacute bacterial endocarditis
b. Repetitive coronary vasospasm d. Non-bacterial endocarditis
c. Embolization to coronary branches
d. Stenosis of coronary ostia 38. Mitral stenosis causes the following
effect on the heart except
32. In hypertensive, heart left ventricular a. Dilatation and hypertrophy of left atrium
hypertrophy is correlated with: b. Dilatation and hypertrophy of left
a. Duration of hypertension ventricles
b. Severity of hypertension c. Dilatation and hypertrophy of right
c. Cause of hypertension ventricle
d. Severity of coronary atherosclerosis d. Dilation of right atrium.

33. In rheumatic heart disease. antibodies 39. Which of the following heart conditions
against the following streptococcal products commonly involved valves of right heart?
are seen in the serum except a. Heart disease in SLE
a. DNAase b. Carcinoid heart disease
b. Streptokinase c. Non-bacterial thrombotic endocarditis
c. Streptolysin S d. Subacute bacterial endocarditis
d. Streptohyaluronidase
CLINICAL CASE
34. Anitschkow cells are believed to the Summary of relevant clinical history
derivative of: Mr. AA aged 45years was a known
a. Cardiac myocyte hypertensive and diabetic for several years,
b. Cardiac histiocyte who was receiving treatment at Amasaman
C. Endocardial smooth muscle cells Hospital. On 21/07/13 his condition became
d. Endothelial cells of lymphatic critical and this was characterized by
difficulty in breathing profuse sweating and
35. In chronic rheumatic heart disease, the restlessness. He was referred to Korle Bu
most common valvular deformities are: Polyclinic where oxygen therapy was started
a. Mitral stenosis and insufficiency but he died shortly.
b. Mitral stenosis alone Autopsy findings
c. Mitral insufficiency alone ● Bilateral pleural effusion
d. Mitral and aortic stenosis combined ● Heavy indurated congested and
oedematous lungs (Right-700g Left-630g).
36. Major criteria in the modified jones' ●Enlarged heart (460g) with left ventricular
criteria include the following except? hypertrophy (ie ventricular wall thickness)
15
ACADEMIC COMMITTEE PBL3B…2025

● Multiple gastric mucosal ulcers 45. A 50-year-old presents with chest pain
●The liver was slightly enlarged (weight whenever he drives his car. He relates that the
1,600g) and shared evidence of early nutmeg pian goes away while he is at the bus stop.
appearance
The pain has not increased in intensity and
● Pancreas was fibrotic
●The kidneys (right 150g and left 160g) frequency over the months. What is the likely
showed finely granular subcapsular surfaces, diagnosis?
a. Stable angina
Use this clinical case to answer Questions b. Unstable angina
40 – 44 below c. Ischemic heart disease
40. What was the immediate cause of death d. Coronary artery spasm
in Mr AA
a. Pulmonary embolism
46. The most frequent cause of aortic valve
b. Congestive heart failure
incompetence and regurgitation is
c. Acute left ventricular failure
a. Latent syphilis
d. Pulmonary oedema
b. Infective endocarditis
c. Rheumatic fever
41. Mr AA had bilateral effusion. What type
d. Aortic dissection
effusion was this?
a Haemorrhagic effusion
47. Physical examination of an asymptomatic
b. Transudate
21-year-old male student with a history of
c. Serous
rheumatic fever during childhood finds an
d. Exudate
early diastolic opening snap with a rumbling
late diastolic murmur and a tapping apex
42. The most likely mechanism for the
beat. The findings are suggestive of
concentric hypertrophy of the left
a. Aortic regurgitation
ventricular wall is?
b. Aortic stenosis
a. Pulmonary hypertension
c. Mitral regurgitation
b. Systemic hypertension
d. Mitral stenosis
c. Pressure overload
d. Pulmonary valve stenosis
48. Which of the following produces a
diastolic murmur
43. What accounted for the heavy lungs in Mr
a. Aortic regurgitation
AA?
b. Aortic stenosis
a. Pulmonary hypertension
c. Mitral regurgitation
b. Pulmonary oedema
d. Supravalvular aortic stenosis
c. Acute left ventricular failure
d. hypertension
49. A 67-year-old male presented to the
Tamale Teaching Hospital 7 weeks post
44. What accounted for the restlessness in
myocardial infarction with complaints
Mr. AA?
recurrent fever and chest pain. Examination
a. Hypoglycemia
of the precordium revealed a pericardial
b. Reduced cerebral perfusion
friction rub. What is the most likely
c. Cerebral oedema
diagnosis?
d. Hypertension
a. Caplan syndrome

16
ACADEMIC COMMITTEE PBL3B…2025

b. Cardiac Tamponade d. Fibrocaseous lesion


c. Takayasu arteritis
d. Dresslers Syndrome 53. The anaemia in Mr SA may explained by
the involvement of which of the following
50. Which of the following is not an organs?
angiotensin-II effects a. Lungs
a. Vasoconstriction c. Kidneys
b. Aldosterone production c. Brain
c. Myocyte hypertrophy d. Liver
d. Renal tubular constriction
54. The most likely diagnosis for the cerebral
CLINICAL CASE manifestation in this case is?
Mr SA aged 56 years with a known a Haemorrhagic stroke
tuberculosis patient who used to receive his b. Ischemic stroke
medication from the Swedru Government c. Cerebral oedema
Hospital. He was admitted to the hospital due d. Tuberculous meningitis
to worsening condition for 2 weeks
associated with loss of consciousness and 55. Mr. SA may also have suffered from
cranial nerve palsy. Four days after which one of the following pericarditis?
discharge, the condition was critical and he a. Adhesive
was referred to the Korle Bu Teaching b. Fibrinous
Hospital where he died on 14/11/13. c. Constrictive
Autopsy findings: d. Purulent
Cachectic and severely dehydrated body of a
middle-aged man with severe pallor mucous 56. Obstructive airway disease is
membranes characterized on pulmonary function testing
The lungs were heavy (Right lung- 780g and by which one of the following?
left lung -670g) both showed cavity more of a. Reduced FEV1/FVC ratio
the upper lobes b. Decreased total lung capacity (TLC)
The kidneys, liver and the brain shown c. Reduced residual volume (RV)
tubercles d. Decreased residual volume/total lung
Use this clinical case to answer Questions capacity
51 -55 below
51. What type of pulmonary tuberculosis was 57. The causes of dyspnoea in a 5-year-old
Mr SA suffering from? child include?
a. Primary pulmonary tuberculosis a. Hyperplastic tonsils
b. Complicated pulmonary tuberculosis
b. Foreign body aspiration
c. Primary progressive pulmonary
tuberculosis c. a and b are true
d. Secondary pulmonary tuberculosis d. All of the above

52. The most commonly gross pathological 58. which of the following pathogens is the
feature diagnostic of Mr SA condition is? main cause of bronchopneumonia?
a. Ghon focus a. influenza virus
b. Hilar lymphadenopathy b. rhinovirus
c. Lung abscess c. Mycoplasma pneumoniae
17
ACADEMIC COMMITTEE PBL3B…2025

d. Enterovirus c. Terbutaline
d. Salmeterol
59. In patients with COPD, long term oxygen
supplementation is prescribed if Pa02 is 64. The following adverse drug reactions is
a. 55mmHg or less not due to the extended pharmacological
b. 65mmHg effect of salbutamol
c. 70mmHg A. Hypokalaemia
d. 5mmHg b. Tachycardia
c. Hyperglycaemia
60. A 54-year-old male smoker is diagnosed d. Hypersensitivity
with chronic bronchitis. A biopsy of the
bronchus is performed. Which of the 65. Which of the following anti-asthma
following is the most likely finding on agents is more likely to cause receptor down
histology? regulation
a. Abundant mucus with plugging of the a. Formoterol
bronchioles b. Theophylline
b. Alveolar destruction and enlargement c. Betamethasone
c. Interstitial fibrosis d. Sodium cromoglycate
d. None of the above
66. Which of the following is not true about
61. A 76-year-old man was rushed to the glucocorticoids Beclomethasone?
pulmonologist at the Tamale Teaching a. they lead to the development of
Hospital with a history of difficulty in tolerance to salmeterol when administered
together`
breathing. After the initial physical
b. reduce bronchial secretion
examination, the specialist made a diagnosis c. presence of B2-receptor upregulation
of bronchiectasis. Which of the following d. reduce bronchial hyperactivity
may have complicated his condition?
67. Which of the following drugs may cause
a. Secondary pulmonary tuberculosis asthma through the lipoxygenase pathway of
b. Pneumonia inflammation?
c. Bronchial carcinoid tumour a. aspirin
d. Fibrocystic Disease of the Pancreas b. propranolol
c. atenolol
62. Which of the following is not a d. paracetamol
bronchodilator
a. Salbutamol 68. The following statement regarding S.
b. Theophylline pneumoniae are true except?
c. Terbutaline a. the most common agent in patients over 40
d. Prednisolone years
b. accounts for 30-50% of cases in adults
63. Which of the following drugs is suitable c. 10-20% of cases in children
as an inhaler for prophylaxis against asthma d. Most common agents in patients who
attack because it highly lipid soluble? are infants
a. Salbutamol
b. Theophylline 69. Cor pulmonale may be caused by
18
ACADEMIC COMMITTEE PBL3B…2025

a. Emphysema c. Preinfarction angina


b. Chronic bronchitis d. Prinzmetal angina
c. Multiple recurrent pulmonary emboli
d. All of the above 75. The most common location for myxoma
of heart is?
70. A person who aspirated a vomitus a. Left ventricle
containing vegetables is likely to develop b. Right ventricle
which of the following c. Left atrium
a. lung abscess d. Interventricular septum
b. Granulomatous inflammation
c. Foreign body resorption 76. Chronic alcoholism is associated with?
d. all of the above a. Hypertrophic cardiomyopathy
b. Dilated cardiomyopathy
71. A 65-year-old man presents to you with c. Restrictive cardiomyopathy
exertional types and dry deterioration with d. Infiltrative cardiomyopathy
hypoxia and cyanosis despite appropriate
medical care. Further evaluation revealed a 77. Most common location for performing
restrictive interstitial lung disease he could be endomyocardial endomyocardial biopsy is?
suffering from a. Right atrium
a. Emphysema b. Right ventricle
b. Chronic bronchitis c. Left atrium
c. Bronchiectasis d. Left ventricle
d. Idiopathic pulmonary fibrosis
78. Which of the following agents play
72. The following are chronic diffuse significant role in the pathogenesis of
interstitial lung diseases except? Centrilobular emphysema?
a. Pulmonary infarction a. Cigarette smoking
b. Cryptogenic organizing pneumonia b. Inactivation of neutrophils
c. Pneumoconiosis c. Inactivation of elastase
d. Diffuse pulmonary fibrosis d. Lung destruction
73. A patient with respiratory symptoms 79. The following findings were obtained
undergoes bronchoalveolar lavage. Which of
from bronchoalveolar lavage in a child with
the following findings would be suggestive
that congestive heart failure is the cause of failure to thrive. Which of these values
the respiratory symptoms? indicated acute bacterial infection?
a. Ciliated bronchial epithelial cells. a. 80% of lavage fluid cells are alveolar
b. Haemosiderin laden macrophages macrophages
c. Rhomboid shaped crystals b. 10-15% T lymphocytes
d. Encapsulated pairs of small vessels c. 5% B lymphocytes and plasma cells
d. 50% neutrophils
74. Calcium Channel Blockers are drugs of
choice for which types of angina? 80. Which of the following differentiate
a. Stable angina small cell undifferentiated lung cancer from
adenocarcinoma of the lung?
b. Unstable angina
19
ACADEMIC COMMITTEE PBL3B…2025

a. It accounts for 25% of lung carcinomas d. The upper extremities thrombosis


b. Strongly associated with smoking
c. It involves the bronchi of the lung 86. Hemarthrosis is defined as?
d. Bloodstream metastasis occur early a. Haemorrhage into the pericardium
disease in about 50.0% of the cases. b. Haemorrhage into the peritoneum
c. Haemorrhage into the thoracic cavity
81. The most important parameter in the
diagnosis of chronic bronchitis d. Haemorrhage into the joints
a. History of prolonged cigarette smoking
b. The Reid index 87. Alpha-antitrypsin deficiency predisposes
c. Productive cough for more than 3 months to which type of emphysema
d. The FEVI:FVC ratio a. Centrilobular emphysema
b. Parasternal emphysema
82. The basic pathological vascular entity in c. Panacinar emphysema
panacinar emphysema is? d. Scar emphysema
a. severe dyspnoea
b. dry cough 88. Extrinsic Allergic Asthma is an example
c. Permanent dilation of the air spaces of?
distal to the terminal bronchiole, usually a. Cell mediated hypersensitivity reaction
b. Type II hypersensitivity reaction
with destruction of lung parenchyma.
c. Type III hypersensitivity reaction
d. Raised pulmonary capillary pressure d. Type I hypersensitivity reaction

83. Which of the following types of 89. Which of the following cause of
atelectasis is sociated with contralateral interstitial pneumonia in patients with
deviation of the trachea member acquired immune deficiency
a. Resorptive syndrome (AIDS) is the odd
b. Compressive a. Cryptococcus neoformans
c. Patchy b. Pneumocystis carinii
d. Contractive c. Histoplasma capsulatum
d. Coccidioides immitis
84. Which of the under listed features of
bronchiectasis is least important? 90. Pulmonary hypertension is elevation of
a. Fusiform dilatation of the bronchioles pulmonary arterial pressure above the upper
b. Saccular dilatation of the bronchioles limit of normal of?
c. Large dilated bronchi near the hilar a. 4mmHg
region b. 14mm Hg
d. The walls of the distended bronchi show c. 10mmHg
inflammation and fibrosis d. 2mmHg

85. Which of the following is rare cause of 91. Haemoptysis in primary tuberculosis may
pulmonary thromboembolism? result from?
a. Thrombosis involving the ilio-femoral A. Red hepatization
b. Thrombosis involving the pelvic venous b. Erosion of pulmonary vasculature
plexus c. Pulmonary congestion
c. Right ventricular mural thrombus d. Chronic cough
20
ACADEMIC COMMITTEE PBL3B…2025

hyperventilation without cyanosis. The pulse


92 Alveolar oedema is a common feature of? was fast (110/min) but regular. Two hours
a. Adult respiratory distress syndrome later the patient returned to the doctor
b. Bronchial asthma complaining that his shortness of breath and
c. Acute left ventricular failure the chest pain mild increased
d. Diffuse pulmonary fibrosis One day after being hospitalized, the patient
complained of severe palpitation. On
93. Healing in a post pneumonic lung is examination the doctor found that the pulse
achieved by? was arrhythmic (irregularly irregular)
a. Adequate perfusion
b. Increased surfactant production
c. Hyperplasia in type two pneumocytes BLOCK 3.3 FEVERS, INFECTIONS
d. Hyperplasia in type one pneumocytes AND INLAMMATIONS II
1. Katayama fever occurs in,
94. With which of the following disorders is a. Egg laid in target organs in liver
jugular vein distention most prominent? b. Egg laid in target organs in skin
a. Abdominal aortic aneurysm c. Egg laid in target organs release
b. Heart failure antibody
c. Myocardial infarction d. Egg laid in target organs release
d. Pneumothorax antigen

95 What is the most common complication of 2. Genitourinary complications of


a myocardial infarction? schistosomiasis include the following
a Cardiogenic shock except
b. Heart failure a. Egg lodge in ureter, urethra cause
C. Arrhythmias lumps and lead to kidney failure
d. Pericarditis b. Egg lodge in ureter, urethra
cause lumps and lead apoptosis
96. Which of the following diagnostic tools c. Egg lodge themselves in wall of
is commonly used to determine the location bladder and develop into polyps
of myocardial damage? d. Eggs block into ovaries, uterus,
a. Cardiac catheter cervix fallopian tube form lump
b. Cardiac enzymes and lead to infertility
C. Echocardiogram
d. Electrocardiogram 3. A 30-year-old drug IV drug abuser get
osteomyelitis, which of the following
bones will be involved?
CLINICAL CASE THREE a. Vertebrae
A man, 28 years of age woke up in the b. Tibia
morning complaining of severe c. Skull
breathlessness and numbness in the fingers. d. Sternum
He also complained of strong pain in the
chest, which relieved in leaning forward. In 4. Surgical debridement is indicated in
the past week he has been down with fever. which osteomyelitis patients?
At a medical consultation, the doctor noticed a. all patients with bone infections
the patient was in respiratory distress with
21
ACADEMIC COMMITTEE PBL3B…2025

b. persistence of fever, local


swelling, and pain after starting 9. Which of the following is the mostly
antibiotic therapy likely organism implicated?
c. like S. aureus acute infection a. Mycoplasma pneumonia
treated with cefazolin b. Streptococcus pneumonia
d. antibiotic therapy started 48 hours c. Haemophilus influenzae
after onset of symptoms d. Staphylococcus aureus

5. Causative agent of acute watery non- 10. Extra-intestinal amoebiasis is caused by


bloody diarrhea excludes.... a. Entamoeba coli
a. Campylobacter jejuni b. Entamoeba histolytica
b. Cryptosporidium hominis c. Entamoeba dispar
c. Norovirus d. Balantidium coli
d. Staphylococcus aureus
11. In lab diagnosis and evaluation of
6. Which of the following is indicative of osteomyelitis, which of the following is
viral and not bacterial meningitis? true?
a. CSF opening pressure 90-200mm a. C-reactive protein does not
H2O, WBC count 10-300/ul correlate with clinical response of
b. CSF opening pressure 200- to therapy
300mm H2O, WBC count 100- b. Elevated ESR does not support
500/ul diagnosis
c. CSF opening pressure 180- c. Wound and abscess culture
300mm H2O, WBC correlates with bone biopsy test
count >100/ul results
d. CSF opening pressure 80-200mm d. Elevated WBC count supports
H2O, WBC count 0-5/ul. diagnosis

7. Classical signs of meningitis include 12. In the epidemiology of osteomyelitis


a. Fever, stiff neck, headache which is false
b. back pain, indigestion, fever a. More common in females
c. rash, inner ear pain, itching b. Overall occurrence is unknown
d. Dry cough, dehydration c. Non hematogenous spread is
more common
d. Long bone osteomyelitis is not
A 15yr old complaints of non-productive common in children
cough associated with fever Temp 37.2°.
Chest examination shows diffuse infiltrates 13. An 8-year-old boy developed a dry
with rattling sounds. Use this to answer 8 and circular, scaly and pruritic lesions on his
9 legs. What is the diagnostic significance
8. Which of the following is the of observing a branching, septate,
manifestation of the above? unpigmented hypae in a KOH/ calcofluor
a. Typical Pneumonia preparation from
b. Atypical pneumonia a. Chromomycosis
c. Aspiration Pneumonia b. Dermatophytosis
d. Fungal Pneumonia c. Eumycetoma
22
ACADEMIC COMMITTEE PBL3B…2025

d. Paracoccidioidomycosis a. Gram positive


b. Corkscrewed shaped bacteria
14. Which of the following about c. Aerobic
paracoccidioidomycosis is incorrect? d. Fastidious
a. A patient is infected after visiting
South America 20. The severity of malaria partly involves
b. The etiologic fungus is dimorphic the production of cytokines by the host
c. The etiologic agent is inherently parasite interaction. Which of the
resistant to amphotericin B following protein is responsible for the
d. The majority of active disease stimulation of the cytokine production by
occurs in men malaria parasites in the host?
a. Chondroitin sulphate
15. Brucellosis is NOT an occasional risk of b. Glycosylphosphatidylinositol
a. Farmers c. Intracellular cell adhesion
b. Doctors molecule
c. Veterinarian d. Tumor necrosis
d. Laboratory personnel
21. The epidemiology of malaria is
16. All these viruses have vaccines for dependent on host characteristics, the
prevention of haemorrhagic fevers except vector and the malaria parasite. Which of
a. Dengue fever virus the following is a correct influence on
b. Ebola zaire malaria epidemiology?
c. Marburg virus a. P. vivax is common in west Africa
d. Yellow fever virus due to the presence of duffy
antigens
17. If you were to produce a vaccine against b. Sickle cell and G6PD are
ebolavirus disease, antigens from which protective from malaria
strains must your vaccine contain c. The longevity of the vector should
a. Tai ebolavirus, Sudan ebolavirus be at most one week to aid
b. Zaire ebolavirus, Sudan sporogony
ebolavirus d. The receptor specificity of the
c. Reston ebolavirus, Sudan parasite determines the species
ebolavirus distribution
d. Sudan ebolavirus, Zaire
ebolavirus, Bundibugyo 22. Which of the following zoonotic illnesses
ebolavirus has no arthropod vector?
a. Plaque
18. The following are bacteria involved in b. Lyme disease
polymicrobial osteomyelitis except c. Brucellosis
a. S. Epidermidis d. Epidemic typhus
b. S. Saprophyticus
c. E. coli 23. The most frequent cause of viral
d. S. agalactiae meningitis is
a. Enteroviruses
19. The following are features of leptospira b. Rhabdoviruses
except, they are c. Herpes simplex 1
23
ACADEMIC COMMITTEE PBL3B…2025

d. Lymphocytic choriomeningitis d. The old world virus presents


virus with renal syndrome

24. Which of the following is not 29. Regarding syphilis, which of the
recommended when doing a work up for following is inaccurate?
patients with suspected meningitis? a. secondary infection forms
a. Prompt lumber puncture condylomata lata.
b. Delay antibiotics until head CT b. Chancre formation occurs in
is done primary infection
c. CBC c. active transmission occurs in
d. LFT tertiary infection
d. Hematogenous spread is observed
25. ln a prolonged neutropenia PUO which of in tertiary infections
the following drug is recommended for
empirical treatment
a. tetracycline 31. Which of the following is most likely
b. chloramphenicol the cause of septicemia?
c. amphotericin a. Pericarditis
d. meropenem b. Soft tissue and mucous
infection
26. The frequent presentation of PUO is c. Diarrhea
a. a common disease manifesting d. Osteomyelitis
in an atypical way
b. a common disease presenting 32. The following statements concerning
typically fever are true except, during fever
c. a rare disease presenting in a. Iron is diverted to the liver
atypical way b. Viral activity is inactivated
d. a rare disease presenting typically c. Effects of endotoxin is
increased
27. The following Ebola virus strains are d. Interferon activity is
known to infect human except enhanced
a. Bombali Ebola virus
b. Bundibugyo Ebola virus
c. Zaire Ebola virus
d. Reston Ebola virus 33. Which of the following statements
concerning the stages and pattern of
28. Hantavirus is an emerging pathogen that fever is inaccurate?
is best described by which of the a. Flush involves vasodilation
following statements b. Prodrome involves mild
a. Influenza-like symptoms are headache, fatigue and general
followed rapidly by chronic malaise
respiratory failure c. Temperature rise involves
b. Has an incubation period of 3-6 generalized shaking with
days chills and feeling of being
c. The new world virus presents cold
with renal syndrome
24
ACADEMIC COMMITTEE PBL3B…2025

d. Defervescence involves c. Blocks inflammatory cells;


termination of sweating Cyclooxygenase pathways
where pain receptors are
released
32. Gram staining was done on the urine d. It is the mediator for painless
sample and it revealed gram positive ulcers
cocci. Which organism can be a cause
of the infection? 36. Sterile pyuria is rarely associated with
a. Staph Aureus
b. Streptococcus aeruginosa a. Urinary tuberculosis
c. Staph epidermidis b. Chlamydial infection
d. Staph saprophyticus c. Fungal infections
d. Pseudomonas infection
33. Which of the following is associated
with chronic diarrhea except? 37. Mr. Adongo is diagnosed with severe
a. Asthma malaria at TTH. Which of the following
b. Celiac disease plasmodium spp. is the most probable
c. Diabetes cause of his malaria?
d. Pneumonia a. Plasmodium falciparum
b. Plasmodium malariae
34. Traveler’s disease is caused by eating c. Plasmodium ovale
contaminated food and drinking water d. Plasmodium vivax
in endemic areas. Which of the
following is not recommended to 38. Cytoadherence of malaria parasites is
someone going to such areas? one of the mechanisms leading to
a. Avoid eating fresh fruits and complicated malaria. Which of the
vegetables unless they can be following is the ligand for
peeled cytoadherence used by plasmodium
b. Avoid taking unpasteurized falciparum?
milk a. pfEMP1
c. You can take ice from tap b. Rifin
water c. Sequestrin
d. Do not brush your teeth with d. Surfin
tap water
39. To diagnose malaria, which of the
35. Inhibition of Sec61 translocation of following approaches is recommended
proteins into the endoplasmic reticulum by world health organization as the gold
by mycolactone results in the following standard?
except, a. Clinical manifestations
a. Alters the junctional b. HRP-2 test
organization and c. LDH and aldolase test
stratification of d. Microscopy
keratinocytes
b. Blocks immune cell 40. Hemolysis of RBCs leads to release of
activation by its which of the following macromolecules
immunomodulatory effect in the circulation?

25
ACADEMIC COMMITTEE PBL3B…2025

a. Porphyrins or crab contaminated with


b. Peptidoglycans miracidi
c. Lipopolysaccharides e.
d. Aminoglycosides 45. Description of schistosoma as blood
fluke is best explained by……..
41. Which of the stage in the life cycle of a. Adult worms residing within
schistosomes is a target for the mesenteric venules and
praziquantel? venous plexus
a. Juvenile schistosomes b. Hematogenous spread from the
b. Adult schistosomes liver to the mesenteric venules by
c. Schistosome eggs schistosoma mansoni
d. Cercariae c. Movement of schistosomula
through the blood to the lungs
42. In the laboratory findings for tropical d. Movement of adult worms from
pulmonary eosinophilia, there is the liver through the blood to the
a. No detectable levels of bladder
antibodies to microfilaria in
circulation 46. A patient reported with dry nocturnal
b. High level of microfilaria in cough, wheezing and dyspnea. Full
the periphery blood count shows 4,000/ul
c. Eosinophil count 450/ul of eosinophils. This best describes……..
blood a. Pulmonary tuberculosis
d. Hyper-IgE levels b. Upper respiratory tract infection
c. Tropical eosinophilia
43. Which of the following is true about d. Aspergilloma
auscultation of tropical pulmonary
eosinophilia (TPU) patient? 47. The urease breath test is requested for
a. Whistling and cracking …………diagnosis
b. Bubbling and rattling a. Treponema pallidum
c. Snoring and wheezing b. Haemophilus ducreyi
d. Wheezing and rattling c. Helicobacter pylori
d. Mycobacterium leprae
44. Which of the following statements
about flukes is matched with the correct 48. Most biological false reaction in VDRL
characteristics? for syphilis diagnosis is related to
a. Blood flukes include species of a. Corynebacterium diphtheria
schistosoma, paragonimus and b. Mycobacterium leprae
fasciola c. Treponema pallidum
b. Sheep liver fluke is d. Helicobacter pylori
hermaphroditic
c. Common to their life cycle is the 49. In tropical pulmonary eosinophilia,
involvement of mollusk as a damage to the lungs is mediated by the
definitive host following except,
d. Clonorchis sinensis is transmitted a. Eiosinophilic cationic protein
by eating raw or undercooked fish from eosinophil degranulation

26
ACADEMIC COMMITTEE PBL3B…2025

b. Complement and opsonization


of microfilaria antibodies 53. In Chancroid, which of the following is
c. IgE mediated hypersensitive not true?
reaction a. the ulcers are painful with rough
d. IgM, IgG mediated ragged edges
hypersensitivity b. caused by haemophilus ducreyi
c. there is a whitish and cheesy
50. In Hansen’s disease, which of the discharge from ulcers
following statements is correct? d. inguinal lymphadenopathy and
a. Highly infectious bubo formation occurs
communicable disease 54. Inflammatory diarrhea is not caused by
b. Tuberculoid leprosy is ……
multibacillary a. giardia lamblia
c. Extensive destruction of the b. entamoeba histolytica
skin and soft tissues with c. shigella dysenteriae
formation of large ulcers d. vibrio cholera
d. Lepromatous leprosy has
abundance of bacilli in 55. Causative agent of acute watery non-
lesions bloody diarrhea excludes….
a. campylobacter jejuni
51. A 20-year-old patient from Central b. cryptosporidium hominis
America presents with single, slowly c. norovirus
expanding non painful scaly lesion on d. staphylococcus aureus
his chest. The lesion is non non-pruritis
and loss of sensation at the site of the 56. As part of meningitis investigations,
lesion. Which of the following disease which of the following is done during
is he likely to have lumbar puncture?
a. Buruli ulcer a. CSF cell count
b. Leprosy b. CSF glucose level
c. Tuberculosis c. CSF pressure
d. Chancroid d. CSF protein

52. In schistosoma clinical manifestations, 57. Aseptic meningitis may be associated


swimmers itch, is best described with……
as……. a. Bacteria
a. cercaria dermatitis, an b. Fungi
inflammatory response at c. Parasites
the site of penetration d. viruses
resulting in itchy rash
b. miracidia dermatitis, an itchy 58. Furious rabies is characterized by the
rash at the site of penetration following except…
resulting in a rash a. Paralysis
c. entry of metaccercaria b. Agitation
through the skin c. Delirium
d. itches in the urethra during d. Hydrophobia
urination
27
ACADEMIC COMMITTEE PBL3B…2025

59. One of the following is predominantly b. Tympanic membrane rupture


caused by viral agents. c. Meniere disease
a. brain abscess d. Wax impaction
b. encephalitis e. External acoustic meatus atresia
c. encephalopathy
d. meningitis 2. Which of the following statement
regarding sensorineural hearing is true?
60. Post-infectious encephalomyelitis is
a. Involves the ear ossicles
commonly caused by……
a. cytomegalovirus b. Easily treatable
b. epstain-barr virus c. Involves the disruption of the
c. varicella-zoster virus conduction pathway from the
d. west-nile virus inner ear to the brain
d. Caused by otitis media
61. Which one is true about post-exposure e. Not the most common type of
prophylaxis following rabid dog bite? hearing loss
a. giving anti rabies vaccine is
enough 3. About Guillain Barre syndrome, the
b. giving anti rabies hyperimmune following are true except?
globulins is standard a. It is a mononeuropathy multiplex
c. giving both anti rabies vaccine
syndrome
and hyperimmune globulins
around the same site of the b. Associated with ascending
inoculation is the standard weakness that can lead to death
d. giving both anti rabies vaccine c. Campylobacter jejuni infection is a
and hyperimmune globulins common risk factor
around the different sites of the d. Miller Fisher syndrome causes
inoculation is the standard paralysis in the eyes
e. There is presence of prickling, pins
62. Risk factors of septicaemia include the and needles sensation in fingers and
following except? toes
a. Age of patient
b. Antimicrobial therapy 4. The following are risk factors for diabetic
c. Decreased immune competence
neuropathy except?
of selected patient population
a. Obesity
d. Increased use of intravascular
procedures b. Smoking
c. Immunocompetence
d. Poor glycaemic control
e. All of the above
3.4 PATHOLOGY OF NERVOUS
SYSTEM 5. Carpal tunnel syndrome
a. Mononeuropathy
1. Causes of conductive hearing loss include b. Polyneuropathy
all of the following except? c. Mononeuropathy multiplex
a. Tympanic membrane perforation d. Major motor function less
28
ACADEMIC COMMITTEE PBL3B…2025

e. None of the above b. Gentamicin


c. Acyclovir
6. Which of the following drugs used in the d. Furosemide
treatment of Multiple sclerosis is given e. Chlorpromazine
orally active?
a. Mitoxantrone 11. The drugs Amitriptyline may be used to
b. Fingolimod
manage vestibular disorders because:
c. Glatiramer
a. it has anticholinergic activity
d. Interferon
e. Natalizumab b. of its antidepressant activity
c. of its antihistaminic activity
7. Parkison’s disease arises from reduction d. of its nonselective blockade of
in which of the following transmitter biogenic amine transporters
systems? e. it reaches the vestibular apparatus
a. Dopaminergic efficiently
b. Serotoninergic
c. Adrenergic 12. The route of administration of gentamicin
d. Peptidergic in Meniere’s disease is
e. Histaminergic a. Intravenous
b. Intramuscular
8. Which of the following about benign c. Subcutaneous
paroxysmal positional vertigo is not d. Intratympanum
correct? e. Rectal
a. It is mainly due to calcium deposits
b. A patient experiences prolong 13. Which of the following antiemetics is a
spinning sensation when they 5HT3 receptor antagonist?
turn their head a. Meclizine
c. A patient may experience nausea b. Promethazine
but not vomiting c. Ondasetron
d. There is no pain in the ear d. Lorazepam
e. No loss of hearing e. Metoclopramide

9. Which of the following classes of drugs 14. The following antiemetic are dopamine
are known to be ototoxic? receptor antagonists except?
a. Penicillins a. Metoclopramide
b. Cephalosporins b. Perphenazine
c. Aminoglycosides c. Promethazine
d. Fluoroquinolones d. Prochlorperazine
e. Phenothiazines e. Granisetron

10. Which of the following is not used in the 15. A 50-year-old male presents with
treatment of vestibular diseases? headaches, vomiting, and weakness of his
a. Dexamethasone left side. Physical examination reveals his

29
ACADEMIC COMMITTEE PBL3B…2025

right eye to be pointing “down and out” 17. Hypertension is most closely related to
along with ptosis of his right eyelid. His the formation of which one of the
right pupil is fixed and dilated and does following types of aneurysms?
not respond to accommodation. Marked a. Berry aneurysm
weakness is found in his left arm and leg. b. Atherosclerotic aneurysm
Swelling of the optic disk (papilledema) c. Mycotic aneurysm
is found during examination of his retina. d. Charcot-Bouchard aneurysm
Which one of the following is most likely e. Saccular aneurysm
present in this individual?
a. Aneurysm of the vertebrobasilar 18. Laminar necrosis and watershed infarcts
artery are most suggestive of
b. Arteriovenous malformation a. Shock
involving the anterior cerebral b. Hypertension
artery c. Fat emboli
c. Subfalcine herniation d. Vascular thrombosis
d. Tonsillar herniation e. Venous sinus thrombosis
e. Uncal herniation
19. A 69-year-old male in an underdeveloped
16. An 18-year-old male high school baseball country develops changes in his mental
player gets hit in the head with a fastball status along with ataxia, deformed knees
in the temporal area. He does not lose and ankles, and an abnormal gait during
consciousness, but afterward develops a which he slaps his feet as he walks.
slight headache. He is not taken to the Physical examination reveals decreased
emergency room. By evening he vibration and proprioception in lower
develops severe headache with vomiting extremities along with absent pupillary
and confusion. At that time he is taken to light reflexes with normal
the emergency room, where, after being accommodation. These signs are most
examined by a neurosurgeon, he is taken likely due to
to the operating room for immediate a. Cysticercosis
surgery for an epidural hematoma. Which b. Neurosyphilis
one of the following is most likely present c. Poliomyelitis
in this individual? d. Rabies
a. Transection of a branch of the e. Progressive multifocal
middle meningeal artery leukoencephalopathy
b. Bleeding from torn bridging veins
c. Rupture of a preexisting berry 20. A lumbar puncture is performed on a
aneurysm patient with headaches, photophobia,
d. Rupture of an arteriovenous clouding of consciousness, and neck
malformation stiffness. If these symptoms are the result
e. Cortical bleeding occurring of bacterial infection of the meninges,
opposite the point of a traumatic then what would examination of the
injury cerebrospinal fluid (CSF) most likely

30
ACADEMIC COMMITTEE PBL3B…2025

reveal? Gross Pressure Appearance a. Schwannoma


Protein Glucose Inflammation b. Medulloblastoma
a. Increased Cloudy Increased c. Oligodendroglioma
Decreased Neutrophils d. Glioblastoma multiforme
b. Increased Clear Increased Normal e. Ependymoma
Lymphocytes
c. Increased Clear Increased Normal 24. An upper motor neuron (UMN) lesion
Mononuclear cells involving cranial nerve VII would most
d. Decreased Clear Decreased Normal likely produce
Lymphocytes a. Contralateral weakness of the
e. Increased Clear Increased Normal lower half of the face with
Mixed sparing of the upper half of the
face
21. A 5-year-old boy presents with projectile b. Decreased gag (pharyngeal) reflex
vomiting and progressive ataxia. Workup with decreased taste sensation from
finds obstructive hydrocephalus due to an the posterior one-third of the
infiltrative tumor originating in the tongue
cerebellum. What is the most likely c. Hemianesthesia of the face with
diagnosis for this cerebellar tumor? flaccid paralysis of the muscles of
a. Ependymoma mastication
b. Glioblastoma multiforme d. Ipsilateral anosmia with primary
c. Medulloblastoma amenorrhea in females
d. Oligodendroglioma e. Ipsilateral paralysis of the soft
e. Schwannoma palate with hoarseness and
dysphagia
22. A 55-year-old woman is suspected to
have a brain tumor because of the onset 25. After recovering from a viral respiratory
of seizure activity. Computed tract infection, a 23-year-old female
tomography (CT) scans and skull x-rays presents with weakness in her distal
demonstrate a mass in the right cerebral extremities that rapidly ascends to
hemisphere that is markedly calcific. This involve proximal muscles. Physical
tumor most likely originated from examination reveals absent deep tendon
a. Astrocytes reflexes, and a lumbar puncture reveals
b. Microglial cells the CSF protein to be increased, but very
c. Ependymal cells few cells are present. A biopsy of a
d. Oligodendrogliocytes peripheral nerve reveals inflammation
e. Schwann cells and demyelination (radiculoneuropathy).
What is the best diagnosis?
23. Which of the following tumors is a. Brown-Séquard’s syndrome
characterized histologically by b. Charcot-Marie-Tooth disease
pseudopalisading, necrosis, endoneural c. Diabetes mellitus
proliferation, hypercellularity, and d. Guillain-Barré syndrome
atypical nuclei? e. Syringomyelia
31
ACADEMIC COMMITTEE PBL3B…2025

d. Weight loss
26. Carpal tunnel syndrome, produced by
damage to or pressure on the median 29. Which of the following statements
nerve deep to the flexor retinaculum, is regarding vestibular neuritis is false?
characterized best by which one of the a. It is commonly associated with
following abnormalities? upper respiratory tract infection
b. It is rare
a. Hyperextension of fingers at
c. It usually lasts from days to weeks
metacarpophalangeal joints and
d. It can be treated with vestibular
flexion at interphalangeal joints suppressants during the first few
(claw hand) days
b. Numbness in fifth finger and of the illness
medial portion of ring finger
c. Pain in thumb, index finger,
30. Which of the following statements
middle finger, and lateral half of
regarding GBS is true?
ring finger
d. Adduction, extension, and internal a. Another name for GBS is chronic
rotation of upper limb (“porter’s inflammatory demyelinating
tip” sign) polyneuropathy
e. Weakness of extensors of wrist and b. The fundamental pathologic
fingers (wristdrop) event in GBS is the stripping of
myelin from axons by
27. Which of the following statements about
macrophages, which occurs in a
Meniere syndrome is false?
a. Meniere syndrome is characterized patchy fashion
by tinnitus, vertigo, and hearing throughout the peripheral
loss nervous system
b. Patients suffering from Meniere c. Several studies have proved that
syndrome can have sudden attacks,
during which they fall to the ground there is a link between a preceding
c. Episodes of vertigo and deafness Shigella dysentery infection and
in Meniere syndrome can last for GBS
hours to a day d. A cardinal feature of GBS is the
d. The deafness associated with
asymmetrical pattern of
Meniere syndrome begins with an
inability involvement
to hear low-frequency sounds and
later progresses to involve higher 31. Which of the following cerebrospinal
frequency sounds fluid findings is NOT characteristic of
viral meningitis?
28. Which of the following is not a long-term a. A protein level of 245 mg/dl
treatment for Meniere syndrome? b. A white blood cell (WBC) count of
a. Salt restriction 80 cells/mm3
b. The Epley maneuver c. A glucose level of 60 mg/dl
c. Diuretics
d. Clear cerebrospinal fluid (CSF)
32
ACADEMIC COMMITTEE PBL3B…2025

c. The bacilli targets Schwann cells of


32. All of the following statements the peripheral verves
concerning cerebrovascular disease are d. The bacilli are not killed by
true except? macrophages
a. The most common cause is e. May present in the third decade of
embolization from cardiac mural life
thrombi
b. Infarction is the most frequent 36. The following statements regarding
cause than haemorrhage tuberculoid leprosy are true except?
c. Haemorrhage is frequently a. Results from a strong B-cell
associated with hypertension response
d. Subarachnoid haemorrhage is b. It is a relatively stable form.
frequently associated with berry c. The lesions are asymmetrical
aneurysm of the circle of Willis d. Never enlarged may be a prominent
e. Atherosclerosis is the most frequent feature
underlying basis of the arterial e. Motor damage is present in some
thrombosis cases.

33. Regarding Mycobacterium leprae, the 37. The clinical presentation of lepromatous
following are true except? leprosy includes the following except?
a. it is acute bacterial infection. a. Has a strong T-cell immunity and
b. Human is the primary host a poor host response?
c. Infects armadillos b. Has anaesthetic macules papules
d. Has an incubation period of 10 c. Nodules are present on the skin.
years. d. The lesions may be tender
e. The organism has low infectivity e. Anaesthesia of the larynx

34. The most likely route of spread of leprosy 38. Ophthalmoscopic examination of a 5-
is? year-old demonstrates a retinal angioma.
a. Haematogenous This finding should raise the possibility
b. Lymphogenous of which syndrome?
c. Direct contact with secretions a. Neurofibromatosis 1
b. Neurofibromatosis 2
from an infected person.
c. Sturge Weber's syndrome
d. Inhalation of secretions from the
d. Von Hippel Lindau syndrome
nostrils of an infected person
e. All of the above 39. Which of the following is filled with
aqueous humor and present at the
35. Which of the following clinical features junction of cornea and iris?
of leprosy is false? a. Limbus
a. Majority of the victims develop b. Lenses
acute signs and symptoms c. Eyebrows
b. The pathogenesis may be cell d. Anterior chamber
mediated
33
ACADEMIC COMMITTEE PBL3B…2025

40. A child develops tumour of the d. None of the above


cerebellum. Biopsy reveals both
neuronal and glial differentiation. Which 45. Which of the following sites does
of the following is most likely the myxopapillary ependymoma frequently
diagnosis? occur?
a. Astrocytoma a. Cerebellum
b. Glioblastoma Multiforme b. Conus medularis
c. Medulloblastoma c. 4th Ventricle
d. None of the above d. Lateral ventricle

41. This is important in memory storage and 46. What is the most severe type of hearing
retrieval lost you can get from cerumen
a. Dopaminergic impaction?
b. Serotonergic a. Hyperaccusia
c. Cholinergic b. Moderate conductive
d. Adrenergic c. Profound conductive
d. Severe conductive
42. A 21yr old female Asian immigrant
presents with complaints of malaise, 47. A 64-year-old man begins to show
fever, arm pains, loss of appetite and behavioral changes and instability, and is
visual problems. Her mother stated that found wandering in the park near his
she fainted a week ago. The physician home. On neurological, there is evidence
cannot palpate a pulse in her lower of mild aphasia and cognitive
extremities and also note that the radial dysfunction, but motor function is
pulses are weak. Her ESR is elevated. preserved. CT scan of the head
What is the most likely diagnosis? demonstrate atrophy of the cortex of the
a. Buerger’s disease frontal lobes. Which of the following is
b. Kawasaki’s disease the most likely diagnosis?
c. Takayasu arteritis a. Alzheimer's disease
d. Thrombophlebitis b. Friedreich's ataxia
c. Huntington's disease
43. A 45yr old man with a large d. Pick's disease
meningioma compresses his brain. A
head CT scan shows a subfalcine 48. A 71- year-old has been in excellent
herniation. This herniation would most health, and practicing competently as an
likely destroy which structure attorney. He is brought to the emergency
a. Cerebella tonsils room following a motor vehicle
b. Cingulate gyrus accident. A workup, including imaging
c. Media temporal lobe of spine, thorax and head is negative, but
d. Medulla the patient is admitted for overnight
observation. His injuries include severe
44. Regarding the bacteria Mycobacterium a. Alzheimer's disease
Leprae b. Friedreich's ataxia
a. Causes acute bacterial infection c. Huntington's disease
b. Human being is the primary host d. Subdural haematoma
c. Affects armadillo
34
ACADEMIC COMMITTEE PBL3B…2025

49. A 1-year-old boy with mental retardation 2. Behavioural interventions for pain are
seizures and facial angiofibromas based on
develops repeated episodes of syncope. A. Motivational techniques
ECG reveals a mass in the left ventricle
B. Operant conditioning
producing intermittent obstruction.
Pathologic exams of the mass show C. Cognitive challenge
cardiac rhabdomyoma. Which of the D. Self-management techniques
following would the patient also have?
a. Acoustic neuromas 3. The goal of Cognitive Behavioural
b. Berry aneurysm Therapy for pain is to help patients to
c. Cortical tubers A. Identify strategies to manage their pain,
d. Meningioma
emotional distress and psychosocial
50. Which of the following causes malignant difficulties
tumours in adults? B. Identify the relationship between their
a. Medulloblastoma thoughts, emotions and behaviour
b. Glioblastoma multiforme C. Alter their beliefs that their problems are
c. Meningitis unmanageable
d. Astrocytoma D. All of the above
51. A 20-year-old male with no history of
hypertension demonstrated a mix of both 4. The gate control model of pain (Mezack &
parenchymal and subarachnoid Wall, 1965) is a
hemorrhage. Parenchymal hemorrhage is A. Psychobiological model of pain
centered on one hemisphere. What is the B. Biological model of pain
most likely source of hemorrhage? C. Psychological model of pain
a. Berry aneurysm
D. Sociological model of pain
b. Charcot- Bouchard
c. Bridging veins
d. Arteriovenous malformation 5. According to the gate control model of
pain, activation of which fibers close the
gate?
BLOCK 3.5 PATHOPHYSIOLOGY OF
PAIN A. C fibers
B. A beta fibers
1. Neuropathic pain is C. T fibers
A. Not distinctly different from nonciceptive D. All of the above
pain
B. Due in part to anatomic and 6. Which of the following is one theory of
biochemical changes in the nervous system
how the placebo works?
C. Seen immediately after most traumatic
injuries. A. Classical conditioning
D. Is sustained by the normal processing of B. Operant conditioning
sensory input by the peripheral or central C. Cognitive challenge
nervous system. D. Self-management techniques

35
ACADEMIC COMMITTEE PBL3B…2025

7. The bio-psychosocial framework is best


described as the study of 12. The increase in length of the radius results
A. Genes to understand mental illness from
B. An individual psychological background A. Epiphysis
C. An individual social and cultural B. Physis
background in order to understand and treat C. Diaphysis
their illnesses D. Metaphysis
D. The interaction of the biological,
sociocultural and psychological influences 13. The aetiology of osteochondrosis include
in the diagnosis of illnesses and all the following except
management A. Trauma
B. Microtrauma
8. How long does pain have to toe considered C. Hereditary
chronic? D. Infections
A. More than 1 to 3 months
B. More than 2 to 4 months 14. Legg-Calve-Perthes disease; all are true
C. More than 3 to 6 months except
D. More than 6 to 12 months A. Typically presents between 4 to 8years
B. Affects more boys than girls
9. Biological factors affecting pain include C. Poor prognostic factors include greater
the following except extent of femoral head deformity
A. Sex D. Age of onset is usually around 13 to 16
B. Age years
C. Disaster
D. Illness 15. Sprain is
A Tear of tendons
10. Nonpharmacologic therapies B. Tear of muscles
A. Can detract from pharmacologic treatment C Tear of ligaments
in cancer patients. D. Contusion of the muscle
B. Often are underused in acute pain
C. Should be considered only in chronic 16. Ankle stress test with the joint in
nonmalignant pain. inversion (calcaneo-fibular ligament);
D. Can induce a number of opioid-like side eversion-deltoid.
effects. A. Anterior talocalcaneal ligament
B. Deep deltoid ligament
11.The formation of the bones of the skull C. Superficial deltoid ligament
A. Intramembranous ossification D. None of the above
B. Membranous ossification
C. Endochondral ossification 17. In plantar fasciitis the pain is typically
D. Cartilaginous ossification located in the
36
ACADEMIC COMMITTEE PBL3B…2025

A. Hind foot A. Swollen DIP joint


B. Mid foot B. Swollen PIP joint
C. Fore foot C. Swollen MCP joint
D. Toes D. Swollen nail pulp

18. Jones fracture is 24. Concerning the Bouchard’s nodes


A. A stress fracture of the 5th metatarsal A. Swollen DIP join
B. An avulsion fracture of the 5th B. Swollen PIP joint
metatarsal C. Swollen MCP joint
C. A stress fracture of the 4th metatarsal D. Swollen nail pulp
D. A stress fracture of the calcaneus
25. Concerning rheumatoid arthritis all are
19. Joints of the wrist include except true except.
A. Distal radioulnar joint A. Swan neck deformity
B. Radiocarpal joint B. Ulnar deviation of the MCP joints
C. Carpometacarpal joints C. Nodules along synovial sheath
D. Proximal radioulnar joints D. Deformity is unilateral

20. Movements about the wrist include 26. Concerning carpal tunnel syndrome, the
except following are true except
A. Flexion A. Compression of the median nerve
B. Ulnar deviation B. The thenar muscles are wasted
C. Radial deviation C. The hypothenar muscles are wasted
D. Medial deviation D. Paresthesia in the distribution of the
median nerve
21. On flexion of the fingers at the PIP joint,
all the fingers point towards the scaphoid. 27. Concerning the valgus ankle stress test
This sign is referred to as the A. Asses the lateral collateral ligaments
A. Finkelstein sign B. Asses the medial collateral ligaments
B. Tinel sign C. Asses the plantar fascia
C. Phallen’s sign D. Asses the Achilles tendon
D. Cascade sign
28. Concerning the varus ankle stress test
22. In the Boutonnieres deformity A. the lateral collateral ligaments
A. Extension of the PIP B. Asses the medial collateral ligaments
B. Flexion of the PIP C. Asses the plantar fascia
C. Extension of the DIP D. Asses the Achilles tendon
D. Seen in rheumatoid arthritis
29. Concerning anterior dislocation of the
23. Concerning Heberden’s nodes shoulder joint the following are true except
37
ACADEMIC COMMITTEE PBL3B…2025

A. It's the commonest type of dislocation A. III


B. The shoulder is held in abduction B. 1
C. The shoulder is held in adduction C. II
D. The shoulder is held in external rotation D. IV

30. The rotator cuff muscles include the 35. The following are pulmonary
following except manifestations of rheumatoid arthritis,
A. Deltoid except?
B. Supraspinatus A. Lung abscess
C. Infraspinatus B. Pleurisy
D. Teres minor C. Pleural effusion
D. Interstitial fibrosis
31. Which of the following ligaments is not
stabilizer of the hip 36. Which of the following is a poor
A. Iliofemoral ligament prognostic factor in rheumatoid arthritis?
B. Ischiofemoral ligament A. Rheumatoid nodules
C. Pubofemoral ligament B. Onset of disease after 60 years
D. Lateral collateral ligament C. Decreased levels of rheumatoid factor
D. Pleurisy
32. Rheumatoid arthritis commonly involves
which of the following joints? 37. The differential diagnosis of Ankylosing
A. Hip spondylitis include the following, except?
B. Ankle A. Psoriatic arthritis
C. Knee B. Crohn's disease
D. Shoulder C. Osteoarthrosis
D. Reiter's syndrome
33 Which of the statements is true regarding
immune basis as the cause of rheumatoid 38. Which of the under listed diseases or
arthritis? syndromes have an infectious particle as the
A. Rheumatoid factor is always detectable in causative agent?
the plasma of patients A. Psoriasis
B. Immune complexes are found in the B. Reiter's syndrome
plasma of patients C. Still disease
C. Immune complexes are elevated in D. Felty's syndrome
chronic disease state
D. Immune complexes are decreased in 39. The following structures contribute to the
chronic disease state formation of knee joints except....
A. Condyle of the tibia
34. Rheumatoid arthritis is an example of B. Head of the fibula
which type of hypersensitive reaction? C. Medial femoral condyle
38
ACADEMIC COMMITTEE PBL3B…2025

D. Lateral femoral condyle B. Arthritis


C. Pencil-in-cup deformity
40. The following are monoarticular except? D. Conjunctivitis
A. Gouty arthritis
B. Osteoarthritis 46. Rheumatoid factor is positive in which of
C. Rheumatoid arthritis the following conditions?
D. Ankylosing spondylitis A. Ankylosing spondylitis
B. Gouty arthritis
41. The following joints diseases commonly C. Osteoarthritis
involve the hands and feet except? D. Felty's syndrome
A. Osteoarthritis
B. Gouty arthritis 47. The following are true of the clinical
C. Psoriatic arthritis presentation of osteoarthritis in a 45-year-old
D. Reiter's syndrome woman, except?
A. Joint stiffness
42. Which of the following joint diseases are B. Monoarticular involvement and
associated with bone erosions? swelling of affected joints
A. Gouty arthritis C. Crepitus
B. Psoriatic arthritis D. Heberden's nodes
C. Reiter's syndrome
D. Juvenile rheumatoid arthritis 48. The following are implicated in the
pathogenesis of osteoarthritis except
43. Which of the following diseases A Ground substance content
commonly affect adults? B. The quality of collagen
A. Osteosarcoma C. Increased activity collagenase
B. Ewing's sarcoma D. Decrease activity of proteoglycanase
C. Chondrosarcoma
D. a and c 49. Which of these neoplastic bone lesions
have genetic basis?
44. The following are extra-articular A Ewing's sarcoma
manifestation of Ankylosing spondylitis B. Osteosarcoma
except? C Chondroma
A. Incompetence aortic valve
D. All of the above
B. Aortic dissection
C. Iridocyclitis
50. The following is or are true of
D. Urethritis
chondroblastoma?
A. It is a well-defined lesion radiologically
45. The following are clinical manifestations
B. Common in persons aged 35 years
of Reiter's syndrome except?
C. Common in females
A. Urethritis
D. Commonly involve all bones
39
ACADEMIC COMMITTEE PBL3B…2025

A. It may be painful
51. Unhappy triad consists of injuries to B. It's fluctuant nature
which of the following anatomical C. It is boggy on palpation
structures? D. It's radiological appearance
A. Anterior cruciate ligament, Medial
collateral ligament, and medial Meniscus 56. The following features regarding a
B. Posterior cruciate ligament, medial and disease joint may be observed in a plain X-
lateral meniscus ray of the joint Except?
C. Lateral collateral ligament., posterior A. Calcifications
cruciate ligament, and anterior cruciate B. A decrease in the joint space
ligament. C. Loss of the bone content
D. Anterior and posterior cruciate ligament D. Increase density of the bone

52. The following is or are clinical features of 57. The following pathological processes are
Still's disease. associated with increase joint space except?
A. The peak age is 30 to 50 years A. Acute osteoarthritis
B. Low grade fever B. Trauma
C. Leukocytosis C. Synovia hyperplasia
D. All of the above D. Foreign body

53. The following statements regarding 58. Which of the following cells are involved
osteosarcoma is or are false? in bone production?
A. It commonly affect diaphyseal region A. Fibroblasts
bones of long B. Osteoclasts
B. It commonly arises in the periosteum of C. Osteocytes
long bones D. Osteoblasts
C. It commonly involved the metaphysial
region of short bones 59. The following statements about
D. All of the above osteoporosis are true except?
A. There is a decrease in the total mass of
54. Which of the following clinical features bone without other structural abnormalities
is common in both arthritis and arthralgia? B. There is normal bone formation
A. Restriction in the range of movements C. Osteoporosis represents a form of bone
B. Swelling pathology
C. Tendermess D. Affects the cortex of the involved bone
D. Erythema commonly

55. The following are helpful diagnostic 60. Regarding Senile osteoporosis which of
features of a neoplastic lesion of a joint the following statement is false?
except?
40
ACADEMIC COMMITTEE PBL3B…2025

A. Associated with increased osteoblastic B. Anti-CCP antibodies


activity C. ANCA (antineutrophil cytoplasmic
B. May be initiated or maintained by a variety antibodies)
of bone cytokines. D. C-reactive protein
C. May be genetically predetermined by the
polymorphisms of the Vitamin D receptor 64. The following are characteristic of
gene inflammatory arthritis, EXCEPT
D. Common in females A. It is associated with soft tissue swelling
and effusion.
61. The following clinical signs can be found B. Morning stiffness lasts around 15
in the hands of a patient with rheumatoid minutes.
arthritis, EXCEPT? C. Systemic lupus erythematosus is a cause.
A. Swan-neck deformity D. The joint pain is worst during the morning.
B. Swelling of the distal interphalangeal
joints. 65. The following are causes of inflammatory
C. Ulnar deviation of the polyarthritis, EXCEPT
metacarpophalangeal joints A. Ankylosing spondylitis
D. Volar subluxation of the B. Juvenile arthritis
metacarpophalangeal joints C. Osteoarthritis
D. Sjogren's syndrome
62. A 48-year-old woman was recently
diagnosed with Rheumatoid arthritis by her 66. A 36-year-old woman has noticed
doctor and sent to the hospital for further swollen fingers and tight skin, which limit
management. She has significantly swelling flexion and extension of the fingers. She also
of the hands. The following will be possible has abdominal discomfort. She was
findings concerning her condition, EXCEPT diagnosed with Systemic sclerosis. Which
A. ESR of 90mm/hr. part of the gastrointestinal (GI) tract is most
B. Positive Anti-CCP antibodies. frequently involved in this condition?
C. Positive anti-ds DNA antibodies. A. Colon
D. Ulnar deviation of the hands. B. Duodenum
C. lleum
63. A young woman presents with a facial D. Oesophagus
"butterfly' skin rash, arthralgias, and fatigue.
On examination her heart and lungs are 67. A 29-year-old woman develops painful
normal, the wrists are swollen and tender on swelling of both hands. She is also very stiff
palpation. Systemic lupus erythematous is in the morning. Physical examination reveals
considered. Which of the following is the symmetrical involvement of the proximal
most appropriate initial test to confirm the interphalangeal joints and
diagnosis? metacarpophalangeal (MCP) joints. Her CCP
A. Anti-double-stranded (ds) DNA antibodies are positive, and ANA is negative.
41
ACADEMIC COMMITTEE PBL3B…2025

Which of the following is the most likely B. Randomized control trials


diagnosis? C. Case control studies
A. Rheumatoid arthritis D. Retrospective studies
B. Systemic sclerosis
C. Systemic lupus erythematosus 73. Which of the following is critical in
D. Osteoarthritis selecting a journal article for current
evidence?
68. Features of musculoskeletal conditions A. Date of publication
include the following, EXCEPT B. Language of publication
A. Polyarticular joint pain. C. Number of authors
B. Interphalangeal joint swelling. D. Length of article
C. Localized organic symptoms.
D. Muscle stiffness. 74 In critical evaluation of a clinical research
article the following should be looked out for
69. Chronic polyarticular joint pain include earlier such as in the introduction section
the following, EXCEPT. A. Intention to treat analysis
A. Commonly associated with systemic B. Sample size
symptoms. C. Confidence interval
B. Duration of more than six weeks D. Hypothesis to be tested
C. It is a sign of self-limited disease.
D. Systemic lupus erythematous is a cause. 75. In the critical evaluation of a clinical
research article which one of the following is
70. Extraarticular manifestations of a means of eliminating bias?
rheumatological diseases include the A. Analysis by Intention to Treat Analysis
following, EXCEPT. B. Determination of sample size
A. Oral ulcers C. Determination of the significance level
B. Raynaud's phenomenon D. Determination of the confidence interval
C. Splinter haemorrhage
D. Synovial swelling 76. Which of the following measurements
would you consider as continuous data?
71. Which of the following research designs A. Age in years
provides the strongest evidence? B. Sex: Female, Male
A. Randomised control trials C. Pain: Mild, Moderate, Severe
B. Meta-analysis D. Events: 10, 20, 30, 40, 50, 60
C. Case control studies
D. Prospective observational studies 77. If the statistical significance is set at
p<0.05, the Alternative Hypothesis is upheld
72. What type of study is most appropriate for when p is
evaluating causality? A. 0.005
A. Meta-analysis of cohort studies B. 0.060
42
ACADEMIC COMMITTEE PBL3B…2025

C. 0.025
D. 0.001 83. Which of the following pains scale is
appropriate for the assessment of chronic
78. In a clinical trial, the following drugs pain
(analgesic effect size, significance) were A. McGill Pain Questionnaire
observed. Which one will you choose to B. Visual analogue scale
manage pain? C. Numeric pain scale
A. Amol (5, p<0.001) D. Multidimensional pain inventory
B. Bmol (20, p=0.04)
C. Cmol (30, p=0.45) 84. Which of the following is not right about
D. Dmol (50, p=0.007) tension headache?
A. Is mostly due to stress/ depression
79. Which of the following analgesic may B. Amitriptyline may be used
cause addiction upon chronic use? C. Is never associated with syncope
A. Ketorolac D. Photophobia may be a symptom
B. Celecoxib
C. Oxycodone 85. Migraine
D. Pregabalin A. Occurs more in males than in females
B. Has the aura phase always associated
80. The following is not a means of visual disturbance
eliminating bias in clinical trial C. It's a secondary headache with
A. Power calculation D. Is not never associated with the menstrual
B. Randomization cycle
C. Blinding
D. Definition of outcome measure 86. Which one of the following is not an
example of primary headache?
81. Which of the following is not part of the A. Oral and mandibular headache
list for appraisal of clinical trial paper? B. Migraine
A. Adequate reference list check C. Cluster headache
B. Explicit description of outcomes D. Tension headache
C. Valid outcome measurements
D. Described randomisation method 87. Rheumatoid arthritis:
A. is a degenerative disorder
82. Which one of the following is not part of B. occasionally starts from the knee
the trial profile in a clinical trial research? C. is an inflammatory arthropathy
A. Blinding D. has Herberden nodes as a common feature
B. Numbers
C. Timing and follow-up of randomization 88. Which of the following clinical features
assignment is not present in rheumatoid arthritis?
D. Interventions A Synovitis
43
ACADEMIC COMMITTEE PBL3B…2025

B. Pain B. It is the duration of time at which period


C. Chronic swelling especially of the knee pain is generated.
D. effusions C. It is the point at which a stimulus is
perceived as pain
89. Which of the following is not a useful D. It is the severity of pain one can perceive
investigation used to diagnose a patient with
rheumatoid arthritis? 94. Pain tolerance is influence by the
A. FBC following except?
B. ESR A. Cultural disposition
C. Synovial fluid examination B. Gender
D. Doppler C. Ones level of education
D. Sleep Deprivation
90. The following are most likely to cause
gouty arthritis when given to a patient for 95. Pain sensation is carried by which of
long term treatment. following pathways to the brain
A. Ethacrynic acid A. Corticospinal tract
B. Amiloride B. Neospinothalamic tract
C. Triamterene C. Paleospinothalamic tract.
D. Spironolactone D. Spinothalamic tract

91. Acute pain results in the following 96. The neospinothalamic tract carries
response information to the following centres except?
A. Decreased respiratory rate A. Limbic centre
B. Increased blood flow to the skin B. Midbrain
C. Increased blood flow to the viscera, kidney C. Post central gyrus
and skin D. Thalamus
D. Reduced gastric acid secretion
97. Which of the following tracts to the brain
92. Which of the following forms of pain carries chronic pain?
allows for adaptation? A. Neospinothalamic tract
A. Breakthrough pain B. Paleospinothalamic tract
B. Intermittent pain C. Recticulospinal tract
C. Persistent pain D. Rubrothalamic tract.
D. Recurrent pain
98. According to the gate theory of pain, the
93. Which of the following statement is most pain fibres synapse at?
appropriate regarding pain threshold? A. Anterior horn.
A. It is the duration of time an individual can B. Midbrain
endure pain C. Substantia gelatinosa
D. Thalamus
44
ACADEMIC COMMITTEE PBL3B…2025

4. The following causes respiratory


99. Which of the following analgesic agents alkalosis except?
does not constitute a therapeutic option for a. Severe anaemia
b. Pulmonary embolism
the Pharmacological management of
c. Morphine
neuropathic pain d. Aspirin
A. Celecoxib
B. Carbamazepine 5. Water reabsorption by the kidney is under
C. Amitriptyline the influence of
D. Pregabalin a. Aldosterone
b. Antidiuretic hormone
c. Renin
100. Which of the following opioid analgesic
d. Angiotensin II
agents possesses the highest equianalgesic
potency when administered parenterally 6. Normal fluid and electrolyte balance is
A. Morphine maintained by
B. Hydromorphone a. Aldosterone
C. Fentanyl b. Antidiuretic hormone
D. Pethidine c. Angiotensin II
d. Both a and b
BLOCK 3.6 GENITOURINARY, GIT
AND ENDOCRINE PATHOLOGY
7. The following cells are correctly paired
1. The kidney contributes to acid-base with their corresponding hormones,
balance by except?
a. Secretion of ammonia a. Gonadotrophs… LH
b. Reclamation of bicarbonate b. Corticotrophs… prolactin
c. Increase ketogenesis c. Somatotrophs…GH
d. Decrease carbon dioxide uptake d. Corticotrophs…Endorphins

2. The medical term for cessation of 8. Which of the following statements about
breathing is pituitary gland is true?
a. Dyspnoea a. The posterior lobe forms 90.0% of
b. Apnea the gland by volume
c. Eupnea b. The anterior lobe forms 90.0% of the
d. Tachypnoea gland by volume
c. It produces trophic hormones
3. Given that pH=7.55, PaCO2= 52 mmHg d. B and C are true
and HCO3- = 40 mmHg. What disorder
is likely to occur? 9. Which of the following statements
a. Metabolic acidosis correctly describes the anatomic location
b. Metabolic alkalosis of the hypothalamus?
c. Respiratory acidosis a. It is lateral to the pituitary gland
d. Respiratory alkalosis b. It is anterior to the pituitary gland
c. It is superior to the pituitary gland
d. It is medial to the pituitary gland
45
ACADEMIC COMMITTEE PBL3B…2025

15. The commonest thyroid malignancy in


10. Regarding the functions of the anterior Ghana is
lobe of the pituitary gland, which of the a. Papillary
following hormones is the odd member? b. Follicular
a. ACTH c. Medullary
b. GH d. Anaplastic
c. Vasopressin
d. Endorphin production 16. The following statements about primary
hyperparathyroidism are true except?
11. The pathological condition ascoiated a. It causes hypercalcemia and
with hyperfunctioning anterior pituitary hypophosphatemia
gland is? b. Middle aged are commonly affected
a. Acromegaly c. It is common in females
b. Gigantism d. Occurs in 1-2.5 per 1,00 population
c. Diabetes mellitus
d. Hypertension 17. What is the prevalence of hypospadias in
the general population?
12. The commonest cause of a. 1:300 live births
hyperprolactinemia in adults is b. 1:300 male live births
a. Pregnancy c. 1:500 live male births
b. Adenoma d. 1:200 male live births
c. Renal failure
d. Drug 18. Undescended testis is commonly
associated with which of the following
13. Which of the following statements penile pathologies?
defines primary empty sella syndrome? a. Priapism
a. An empty sella turcica not filled with b. Epispadias
pituitary tissue c. Hypospadias
b. An enlarged empty sella not filled d. Phimosis
with pituitary tissue
c. An enlarged sella turcica filled with 19. Congenital abnormalities of male infants
pituitary adenoma may complicates as?
d. Sella turcica filled with tissue a. Infertility
b. Lower urinary obstruction
14. Which of the following hormones and c. Stricture
syndromes are correctly paired? d. All of the above
a. ACTH hypersecretion by a pituitary
lesion… Cushing’s disease 20. Which of the following involves the
b. ACTH hypersecretion with coronary sulcus and inner membrane of
hyperfunctioning of the adrenal the prepuce?
cortex … Cushing’s disease a. Balanoposthitis
c. ACTH hypersecretion with b. Condyloma acuminatum
hyperfunctioning of the adrenal c. Condyloma lata
cortex … Cushing’s syndrome d. All of the above
d. None of the above

46
ACADEMIC COMMITTEE PBL3B…2025

21. Which of the following conditions 26. The commonest screening method for
involves the shaft of the penis, scrotum prostate cancer in Ghana is estimation of
and the genital region of a man? serum?
a. Bowen’s disease a. PSA density
b. Erythroplasia of Queyrat b. PSA level
c. Condyloma acuminatum c. PSA velocity
d. Bowenoid papillosis d. Age specific PSA range

22. Which of the following serotypes of 27. What is the prevalence of oesophageal
papilloma virus is currently implicated in atresia globally in human population?
squamous cell carcinoma of the penis? a. 1:30,000 live births
a. 6 b. 1:1,500 live births
b. 11 c. 1:500,000 live births
c. 16 d. 1:800 live births
d. 18
28. Which of the following is/are
23. Regarding the clinical features of premalignant lesions of the oesophagus?
cryptorchidism, which of the following a. Paterson-Kelly syndrome
statements is false? b. Achalasia
a. It is bilateral in 35.0% of the cases c. A and B are correct
b. It occurs in 1.0% of full form males d. All of the above
c. It occurs in 4.0% of preterm male 29. Which of the following is/are rare
new born complication(s) of gastro-oesophageal
d. It is common on the right testis reflux disease?
a. Haemorrhage
24. The following pairing of pathology and b. Oesophagitis
the sites are true except? c. Perforation
a. Varicocele…common on the left d. Barrett’s oesophagus
scrotum
b. Cryptorchidism…common on the 30. Which of the following is a significant
right testis gastric protective mechanism or
c. Torsion…common on the right substance?
testis a. Gastric mucus
d. Undescended testis…common on the b. Anatomic integrity of the mucosa
right testis c. Increase blood flow
d. Increase prostaglandin E series
25. The following pairings are true except? synthesis
a. Germ cell tumour…common in the
white population 31. The following complications of
b. Germ cell tumour…common in congenital pyloric stenosis are true
men aged above 50 years except?
c. Seminomas…account for 75% of all a. Active peristalsis
germ cell tumours in man b. Profuse vomiting
d. Yolk sac tumour… elevated level of c. Respiratory alkalosis
serum alpha fetoprotein level d. Dehydration

47
ACADEMIC COMMITTEE PBL3B…2025

32. Which of the following cross-matching is 36. Which of the following statements
false? differentiate between typhoid ulcer from
a. Burns…Curling’s ulcers tuberculous ulcer?
b. Pyloric stenosis…projectile a. The ulcers are located at the
vomiting antimesenteric border of the bowel
c. Nitric oxide synthase b. There is lymphoid system
deficiency…congenital pyloric involvement
stenosis c. There is lymphadenopathy
d. Trisomy 13… congenital pyloric d. All of the above
stenosis
37. Which of the following tissues in the
33. The following regarding cancer of the corpus uterine is/are very sensitive
stomach are true except? oestrogen stimulation?
a. Early cancer…5 year survival is a. Myometrium
50.0% b. Endometrial stromal cells
b. Malignant lymphoma…account for c. Endometrial glands
5.0% of all stomach cancers d. Endometrial stroma cells and
c. Adenocarcinoma…90.0% of all glands
stomach cancers
d. H. pylori…75.0% of stomach 38. The commonest site of endometriosis
cancers externs is?
34. The following statements and the a. Urinary bladder
corresponding pathological conditions b. Ovary
are true except? c. Myometrium
a. Malrotations of GI d. Umbilicus
structures…right retrocecal
appendix 39. The following pairing regarding the age
b. Failure of the endodermal hindgut to range and the pathology are true except?
open at the right a. Leiomyoma…50-65 years
position…imperforate anus b. Leiomyosarcoma… 60-70 years
c. Meckel’s diverticulum…2.0% of the c. Endometrial carcinoma…55-65
population years
d. Constricted portion of the anorectal d. Cervical carcinoma…20-40 years
segment…aganglionic region
40. Which of the following infectious agents
35. The following diseases and the regarding cervicitis is the odd member?
corresponding complications are a. Chlamydia trachomatis
correctly except? b. Escherichia coli
a. Haemorrhoid…prolapse c. Candida albicans
b. Intestinal perforation…congenital d. Trichomonas vaginalis
megacolon
c. Functional intestinal 41. The risk factor commonly implicated in
obstruction…hyperkalaemia the pathogenesis of cervical cancer is?
d. Intestinal obstruction…volvulus a. Multiple sexual partners
b. Infection with human papilloma
virus serotype 18
48
ACADEMIC COMMITTEE PBL3B…2025

c. Frequent coitus 47. The average weight (g) of an adult male


d. Early onset of sexual activity kidney is?
a. 50
42. Which of the following malignancy of the b. 100
vagina is the odd member? c. 200
a. Squamous cell carcinoma d. 250
b. Sarcoma boitryoides
c. Adenocarcinoma 48. The following statements regarding
d. Clear cell carcinoma bilateral renal agenesis is/are true except?
a. Infants have wide set eyes
43. Which of the following statements about b. Infants have broad and flattened nose
squamous cell carcinoma of the vulva is c. Infants have receding chins
false? d. Infants have small and low-set ears
a. It accounts for 15.0% of all vulva
cancers 49. What is the incidence of horseshoe
b. It occurs in women aged over 60 kidney in the general population?
years a. 4.0%
c. High risk human papilloma viruses b. 1.0%
are implicated c. 0.4%
d. All of the above d. 10.0%

44. The following are components of the 50. What is the prevalence of adult polycystic
Stein-Leventhal syndrome except? kidney disease in human population?
a. Excess androgen production a. 1:1000
b. Amenorrhea b. 1:10000
c. Infertility c. 1:25000
d. Virilism d. 1:100000

45. The following is/are true of ovarian 51. Which of the following cystic diseases of
neoplasms the kidney is a risk factor for
a. Malignant lesions spread locally via adenocarcinoma?
peritoneal cavity a. Dialysis associated cystic disease
b. Omental biopsy is required for b. Glomerulocystic disease
staging c. Medullary spongy disease
c. Ascitic fluid cytology is required for d. Uraemic medullary cystic disease
staging
d. All of the above 52. Which of the following aetiological
agents can cause haematuria at all levels
46. Which of the following statement about of the urinary tract?
the risk factors of ovarian cancer is false? a. Malignant hypertension
a. Peutz-Jeghers syndrome b. Osler-Weber Disease
b. Down’s syndrome c. Trauma
c. Lynch syndrome d. Acute pyelonephritis
d. Turners’ syndrome

49
ACADEMIC COMMITTEE PBL3B…2025

53. The presence of which of the following a. Abnormality in some but not all the
cast in routine urinalysis indicates acute glomeruli
kidney injury? b. Only a portion of each individual
a. Hyaline glomerulus is affected
b. Leukocyte c. The entire individual glomerulus is
c. Epithelial affected
d. Red blood cell d. Fibrosis involving only a segment
of the involved glomerulus
54. Which of the following defines nephrotic
syndrome? 60. Type III hypersensitivity reaction is
a. Protein >3.5g/surface area/day implicated in which of the following
b. Protein > 3.5g/1.73𝑚2 /surface/day renal diseases?
c. Protein>3.5g/1.73𝒎𝟐 /day a. Goodpasture’s syndrome
d. Protein > 3.50/1.73𝑚2 /surface/day b. Minimal change disease
c. Membranoproliferative disease
55. Which of the following causes of d. SLE
nephrotic syndrome is the odd member?
a. Uncontrolled DM 61. The commonest congenital cause for
b. Hep B virus infection deficient uptake of bilirubin by the liver
c. HIV infection is?
d. Malaria a. Crigler-Najjar syndrome
b. Gilbert syndrome
56. The following may complicate acute c. Dubin-Johnson syndrome
renal failure except? d. Viral hepatitis
a. Hypertension
b. Fluid retention 62. The main enzyme in the Poloy pathway
c. Polyuria of glycogenetion is called?
d. Uremia a. Aldose reductase
b. Sorbitol reductase
57. which of the following is a rare cause of c. Aldose oxiase
acute renal failure? d. Aldose phosphatase
a. Hepatorenal syndrome
b. Dehydration 63. In diabetic retinopathy, which of the
c. Snake bite following compounds accumulates in the
d. Acute pyelonephritis retina?
a. Glycogen
58. The following statements regarding b. Sorbitol
chronic renal failure is/are true? c. Glucose
a. Nephrons less than 25.0% of the d. Starch
normal
b. Uremia is a common feature 64. Which of the following liver enzymes is
c. A and B are true elevated in biliary duct obstruction?
d. All of the above a. Alanine transaminase
b. Aspartate transaminase
59. The following statement defined focal c. Gamma glutamyl transferase
segmental glomerulosclerosis d. Alkaline phosphatase
50
ACADEMIC COMMITTEE PBL3B…2025

70. Which of the following cells is sensitive


65. Crohn’s disease is characterized by the to radiations and hence may be managed
following histology, except? by radiotherapy?
a. Non-caseation granulomas a. Skeletal muscle cell
b. Superficial mucosal ulcerations b. Testicular cells
c. Stricture formation in chronic cases c. Chondrocytes
d. Widening of submucosa due to d. Osteocytes
oedema
71. Which of the following malignant
66. The persistence of which of the following tumours spread by the transcoelomic
serum markers of hepatitis B virus route?
indicates continued infectivity and likely a. Liver
progression to chronic hepatitis.? b. Gallbladder
a. HbsAg c. Stomach
b. HbeAg d. All the above
c. IgManti-HBcAg
d. AntiHbe 72. The following carcinogens and the
cancers are correctly matched except?
67. The following syndromes are associated a. Polycycline
with extra-colonic manifestations hydrocarbon………Skin cancer
except? b. Vinyl
a. Peutz-Jeghers syndrome chloride…………………Liver
b. Gardener’s syndrome Cancer
c. Turcot’s syndrome c. Aflatoxin
d. Adenomatous polyposis coli B1………..Hepatocellular cancer
d. Fiber
diet…………Gastrointestinal
68. The Epstein-Bar virus (EBV) is cancers
implicated in malignant lesions in the
following cells and tissues except? 73. Which of the following defines familiar
a. Parotid duct epithelium Adenomatous Polyposis?
b. Oropharyngeal squamous a. Gastrointestinal polyps > 100
epithelium b. A minimal100 polyps in the
c. Colonic mucosa colon
d. B-lymphoctes c. Mutations in RB gene
d. Malignant transformation
69. The Human Papilloma Virus (HPV)
serotypes are risk factors for the 74. The APC gene is commonly located on
following neoplastic conditions except? which chromosome?
a. Condyloma acuminatum with a. 5
low cancer risk b. 17
b. Condyloma acuminatum with c. 15
high cancer risk d. 21
c. Bowenoid papulosis
d. Laryngeal carcinoma

51
ACADEMIC COMMITTEE PBL3B…2025

75. The germ-line mutation of the APC gene


usually results in which of the following? 80. An 87yr old woman presented with iron
a. Truncation of the encoded protein deficiency anaemia. The differential
b. Gene amplification diagnosis is?
c. Robertsonian translocation a. Bleeding fibroid fibroids
d. Tumour progression b. Left side adenocarcinoma of the
colon
76. Regarding the malignant transformation c. Right side adenocarcinoma of the
of oedematous colonic polyps, which of colon
the following is true? d. All of the above
a. Adenocarcinoma inevitably develops
in all patients if left unattended, 81. Which of the following causes of renal
usually during 3rd decade of life cancer is the odd member?
b. Adenocarcinoma inevitably develops a. Renal stone
in all patients if left unattended, b. Tuberous sclerosis
usually during 2nd decade of life c. Von Hippel Lindau syndrome
c. Adenocarcinoma inevitably d. Adult polycystic kidney disease
develops in all patients if left
unattended, usually during 4th 82. Which of the following renal
decade of life malignancies has male to female ratio?
d. The number of carcinomas usually a. Renal adenocarcinoma
exceeds 100 b. Transitional carcinoma of renal
pelvis
77. Which of the following best describes c. Chromophobe renal cell
Duke's Stage C colorectal cancer? carcinoma
a. The tumor will invade the mucosa of d. Clear cell sarcoma of kidney
the bowel wall 83. Which of the following renal causes are
b. The tumor invades only the muscular associated with the loss of chromosome
layer of the bowel wall 1?
c. The tumor involves the para-colic a. Renal adenocarcinoma
lymph nodes b. Transitional carcinoma of renal
d. The tumor invades the full thickness of pelvis
the bowel wall c. Chromophobe renal cell
carcinoma
78. A 90-year-old woman with cancer of the d. Clear cell sarcoma of kidney
caecum will most likely present with
a. Intestinal obstruction 84. Beckwith-Wiedemann syndrome/disease
b. Anaemia with abnormalities in which of the
c. Altered bowel habits following chromosomes/genes?
d. Constipation a. WT1
b. WT2
79. Most colon carcinomas are; c. WT3
a. Adenocarcinoma d. Trisomy 18
b. Squamous cell carcinomas
c. Poorly differentiated 85. What proportion of Wilms’ tumours
d. Benign show loss of heterozygosity (LOH) at
52
ACADEMIC COMMITTEE PBL3B…2025

11p13? 91. Which of the following statements


a. 10-15% regarding Pheochromocytoma is true?
b. 25-29% a. It is the most common disorder of
c. 30-40% the adrenal medulla
d. 50% b. Plasma catecholamine levels are
>2000pg/ml
86. Which of the following is the commonest c. The normal urinary metanephrines
cause of ACTH-independent cause of are >1.3mg/d
Cushing’s syndrome in a 64-year-old d. Urinary epinephrine levels are
man? 500ug/d
a. Pituitary gland adenoma
b. Pituitary gland microadenoma 92. Which of the following causes of SIADH
c. Adrenal gland adenoma secretion is the odd number?
d. Ectopic production of cortisol a. Lung abscess
b. Prostate ca
87. Which of the following is a useful c. Oat cell carcinoma of the bronchus
outpatient screening test for Cushing’s d. Mesothelioma
syndrome?
a. Urine analysis 93. Which of the following is/are symptoms
b. Overnight low dose of SIADH secretion?
dexamethasone suppression test a. Confusion
c. Overnight dexamethasone b. Convulsion
suppression test c. Hemiparesis
d. Urine free cortisol d. All of the above

88. The following are or is true of urinary 17- 94. The following are true of diabetes
ketosteroids insipidus except?
a. Testosterone is a component a. It may results from failure of ADH
b. Androstenedione is a component secretion
c. Metabolites of cortisol b. In the great majority of the cases,
d. All of the above the cause is head trauma
c. 30% of cases are idiopathic
89. The normal plasma level of ACTH in d. Permanent cranial diabetes insipidus
adult male is? will occur when over 80% of the
a. 2.2-11.6 pmol/L pathway is destroyed
b. 10-50 pmol/L
c. 415 pmol/L 95. Which of the following causes of
d. 1.2-1.5pmol/l hyperprolactinemia is the odd member?
a. Sleep
90. The most sensitive and specific test for b. Pituitary adenoma
Phaeochromocytoma is? c. Stress
a. Urinary VMA d. Exercise
b. Urinary catecholamines
c. Urinary metanephrines 96. What percentage of the pituitary gland
d. Urinary clonidine must be destroyed for clinical
hypopituitarism to occur?
53
ACADEMIC COMMITTEE PBL3B…2025

a. 75% d. Microcytic normochromic red


b. 50% blood cells
c. 25% e. Microcytic hypochromic red
d. 10-39% blood cells

97. Which of the following factors may 3. Which of the following ratio is true of T-
invalidate thyroid function is the odd cells in bone marrow of a 12 year old
member?
male?
a. Shock
a. CD4+ : CD8+ = 2:1
b. Sepsis
c. Dopamine b. CD4+ : CD8+ =1:2
d. Chronic renal failure c. CD4+ : CD8+ = :1
d. CD4+ : CD8+ = 2:2
98. Undetectable levels of TSH may be due
to the following except? 4. Which of the following is an immune-
a. Toxic nodular goiter related cause of neutropenia?
b. Autoimmune thyroiditis a. Chédiak-Higashi syndrome
c. Exogenous thyroxine overdose b. Lazy leukocyte syndrome
d. Euthyroid c. Chronic granulomatous
disease of childhood
99. Hydrogen ions are excreted d. Felty's syndrome
a. through urine 5. The purpose of a major cross-match in
b. Via rinse
transfusion medicine is to?
c. through the air
a. To identify the donor blood
d. Via the kidney
group
b. To identify the patient's blood
BLOCK 3.7 HAEMATOLOGY group
c. To give safe blood
1. What is the common clinical d. To detect atypical antibodies
presentation of hemophilia in affected that are directed against
individuals? foreign antigens on donor
a. Prolong bleeding time RBCs
b. Petechial haemorrhages
c. Bleeding gum 6. In transfusion medicine, forward blood
d. Hemarthrosis grouping involves?
a. Patient RBCs are added to test
2. Peripheral blood film from a 78year old tubes that contain either anti-
woman diagnosed of adenocarcinoma of A or anti-B test serum.
the colon will show all of the following b. Patient RBCs are added to test
except? tubes that contain exogenous
a. Anisocytosis antigens
b. Poikilocytosis c. No agglutination reaction
c. Normochromic normocytic red
blood cells

54
ACADEMIC COMMITTEE PBL3B…2025

d. Patient serum is added to test b. Haemoglobin


tubes containing either A or B c. Erythropoietin
test RBCs d. Severe anaemia

7. Neural tube is associated with which 13. In a 34 yr old healthy adult, what is the
nutritional anaemia rate of hemopoiesis?
a. Iron a. 10^11 new cells replaced daily
b. vitamin B12 b. 10^13 new cells replaced daily
c. B9 c. 10^12 new cells replaced daily
d. vitamin B6 d. 10^14 new cells replaced daily

8. The concentration of HbA2 in a neonate 14. The commonest Haemoglobin found in


is? cord blood of a baby born with Down
a. 24% syndrome is?
b. 2.5℅ a. 2alpha/2beta globin chains
c. 0.5% b. 2alpha/2delta globin chains
d. 97% c. 2alpha/2gamma globin chains
e. 12% d. 2gamma/2beta globin chains

9. Which of the following transport 15. Which of the following percentages of


proteins is synthesized in the stomach? blast in the bone marrow defines acute
a. Transcobalamines myeloid leukemia?
b. Intrinsic factor a. 30.0%
c. Cobalamines b. 2.0%
d. Cobalophilines c. 10%
d. 15%
10. What percentage of oral iron is absorbed
in the gastrointestinal tract? 16. The clinical features of multiple
a. 5-10% myeloma include?
b. 1-3% a. Bronchiectasis
c. 35-40% b. Restrictive cardiomyopathy
d. 15-25% c. Nephrotic syndrome
d. All of the above
11. The first organ of haematopoiesis in a
foetus is? 17. The following are true of blood group O
a. Bone marrow except?
b. Thymus a. Can be transfused into any
c. Yolk sac patient
d. Kidney b. Blood group O RBC lack A
and B antibodies
12. The hormone that acts as a sensor in c. Can receive only O blood
hematopoiesis in humans is? d. Can be given to a neonate
a. Oxygen
55
ACADEMIC COMMITTEE PBL3B…2025

18. The components of a standard cross- b. Granular cytoplasm


match does not include c. Nucleus may be indented
a. ABO group d. Very motile
b. Rh type
c. Antibody screen for atypical 24. The following are secondary lymphoid
antibodies organs except?
d. Indirect Coombs test a. Spleen
b. Thymus
19. The following are common causes of c. Lymph nodes
haemolytic disease of the newborn d. Peyer’s patches
a. Kell
b. Rh 25. Enlargement of which of the following
c. I organs or in a disease condition tissue
d. MN may lead to anaemia without blood lost
externally?
20. The following pairings are incorrect a. Kidney
except? b. Liver
a. Blood group A patient--- c. Spleen
Duodenal ulcer d. Heart
b. Blood group A patient---
agglutination reaction with AB 26. The following are true of hereditary
donor spherocytosis except?
c. Blood group B donor---anti B in a. It is caused by intrinsic defects
the serum in the protein involved in the
d. Blood group O---less common vertical interaction between the
red cell membrane skeleton and
21. Infectious causes of lymphocytosis does lipid bilayer of the red blood cell
not include? b. It is caused by intrinsic defects
a. Pertussis in the protein involved in the
b. Measles horizontal interaction between
c. Malaria the red cell membrane skeleton
d. Hepatitis and lipid bilayer of the red
blood cell
22. The following are true of T helper cells. c. An autosomal dominant
They are inheritance pattern is seen in
a. CD4- about 75% of cases.
b. CD4+CD8- d. The remaining 25% of the
c. CD4-CD8+ patients have compound
d. CD4- heterozygosity

23. The following are true of mature 27. The following are known examples of
lymphocytes except? qualitative haemoglobinopathies except?
a. Scanty sky blue cytoplasm a. Hb C disease
56
ACADEMIC COMMITTEE PBL3B…2025

b. Hb E disease 33. Which of the following


c. Hb C- thal disease chemotherapeutic agents is BCR-ABL
d. Hb F disease inhibitor?
a. Cyclosphosphamide
28. The following may complicate b. Hydroxyurea
hereditary spherocytosis except? c. Imatinib
a. Severe haemolytic anaemia d. Interferon
b. Mild jaundice
c. Cholelithiasis 34. Which of the following percentages of
d. Moderate splenic enlargement blasts in the bone marrow defines acute
myeloid leukaemia?
29. The following are associated clinical a. 30.0%
features of Cooley’s anaemia except? b. 2.0%
a. It is a disease adulthood c. 10%
b. It is associated with extravascular d. 15%
haemolysis
c. Thinning of the cortical bone 35. The peak age for acute myeloid
d. Growth retardation is a common leukaemia is?
feature a. 70 years
b. 60 years
30. An Hb electrophoresis of a child with c. 3 years
Cooley’s anaemia will show the d. 7 years
following except?
a. Elevated HbF 36. In chronic myeloid leukaemia, the
b. Elevated HbA2 abnormal chromosome is
c. Deceased HbF a. T(9,22)
d. Decreased HbA b. 22
c. 9
31. Which variant of the acute myeloid d. T(9,24)
leukaemia is associated with DIC?
a. AML7 37. The bony lesion that is commonly seen
b. AML3 in multiple myeloma is?
c. AML4e a. Osteoblastic
d. AML6b b. Osteoporotic
c. Osteolytic
32. Which variant of the acute myeloid d. Osteomalacia
leukaemia is associated with Down
syndrome? 38. Which of the following finding is
a. AML7 pathognomic feature of Mycosis
b. AML3 fungoides?
c. AML4e a. Sezary cells
d. AML6b b. Starry sky appearance
c. Pautrier microabscesses
57
ACADEMIC COMMITTEE PBL3B…2025

d. Skin rashes a. Bleeding gums


b. Dental anargic
39. Lymphocytes constitutes what c. Ulcers
percentage of the body’s weight of an d. Dental cysts
adult male?
a. 45% 44. The best investigation to confirm the
b. 24% diagnosis is?
c. 5% a. CT scan of the head
d. 0.5% b. MRI of the head
c. Core needle biopsy
40. Which of the following is the easiest d. Bone marrow aspirate
way to diagnose multiple myeloma at the
primary health care centre? 45. What is the cure rate for Burkitt’s
a. Electrophoresis lymphoma?
b. Clinical history a. 90%
c. X-ray of the bone b. 75%
d. Heating urine collection form c. 67%
the patient d. 60%

41. Pel-Ebstein fever is most likely a clinical 46. The EBV is implicated in what
feature in which of the following proportion of sporadic Burkitt’s
haematological malignancy? lymphoma?
a. Burkitt’s lymphoma a. 25%
b. Nodular sclerosing HL b. 30%
c. ALL c. 45%
d. NHL d. 35%
A 12 year old girl from Nandom was Mr. Kofi a farmer from Kumasi had children
referred to the paediatric surgeon with a with Madam Hawa from Nandom. The elder
rapidly growing mandibular swelling. The daughter Christiana wants to marry Mr.
surgeon made a working diagnosis of NHL, Edmund, the son of her maternal uncle.
most likely Burkitt’s. Use this to answer Druing the marriage counselling process,
questions 42-46. Edmund was told that Christiana’s paternal
grandmother doed of a bleeding disorder
42. What histological variant is the most called haemophilia A. By further interaction
likely?
with Christiana’s mother, Edmund got know
a. Organ transplanted
his grandfather (Mad Hawa’s father) also
b. HIV associated
died of haemophlia A. Mr Kofi was the only
c. Endemic
d. Sporadic surviving twin male child of his parents, the
other twin died at birth. Christiana is
43. The most likely oral presentation will however asymptomatic. This kept Edmund
be? thinking for he had already had four children

58
ACADEMIC COMMITTEE PBL3B…2025

with Christiana while in school. Use this to a. PTT


answer questions 47-51. b. PT
c. Factor assay
47. What id the probability that Mr. Kofi d. Clinical history
would have haemophilia A?
a. ½ 53. Which of the following viruses is
b. ¼ implicated in the aetiology of aplastic
c. 1 anaemia in human?
d. 1/8 a. HIV
b. HBV
48. What percentage of Mr. Kofi’s sisters c. Parvovirus B19
had clinical disease? d. HPV
a. 50%
b. 25% 54. Which of the following defines sickle
c. 75% cell disease?
d. None of the above a. Genotype AS
b. Genotype SS
49. What percentage of Mr. Kofi and Mad c. Genotype SC
Hawa’s children will express the d. Genotype SB-thal
disease?
a. 50% 55. The protein from food which first binds
b. 25% and transport vitamin B12 in the upper
c. 75% gastrointestinal tract is?
d. None of the above a. Transcobalamines
b. Intrinsic factor
50. Karyotyping of Edmund mother’s c. Cobalamines
chromosome revealed trisomy 21 and no d. Cobalaphilines
other abnormality. What is the
probability that Edmund’s sons will have 56. Oral iron absorption maximally occurs
the disease? in the?
a. ½ a. Stomach
b. ¼ b. Jejunum
c. 1/8 c. First part of the duodenum
d. None of the above d. Ileum

51. What is the probability that Edmund’s 57. Which of the following red blood cell
daughter will be normal? indices is commonly used in
a. ½ morphological classification of anaemia
b. ¼ in humans?
c. 2/3 a. Hb
d. None of the above b. MCH
c. MCHC
52. How is Haemophilia A diagnosed? d. MCV
59
ACADEMIC COMMITTEE PBL3B…2025

58. The following are cofactors in


erythropoiesis except?
a. Folate
b. Manganese
c. Cobalt
d. Calcium
The figure above is a bone marrow aspirate
59. In a 72.0 kg male, what percentage of from a 12-year-old boy. Use it to answer
haemopoiesis activity takes place in the questions 63-66.
bone marrow?
a. 25% 63. The picture shows which type of acute
b. 50% leukaemia in the FAB stage?
c. 75% a. ALL2
d. 100% b. AML1
c. AML2
60. Which of the following growth factors d. AML3
act on pluripotent cells?
a. IL-1 64. This condition is commoner in which
b. TNF age group?
c. SCF a. Children
d. IL-3 b. Young adults
c. Adults
61. Weibel-Palade bodies are synthesized in d. Extremes of age
which of the following cells?
a. Magakaryocytes 65. The common cell types in this condition
b. Platelets is?
c. Liver cells a. B lymphocytes
d. Endothelial cells b. T lymphocytes
c. Marginal cells
62. What is the normal life span of platelets d. Mantle cells
in peripheral blood?
a. 100-120 days 66. The prognosis of this condition is?
b. 90-100 days a. Good
c. 9-10 days b. Very good
d. 1-5 days c. Bad
d. Excellent

The Table shows the peripheral blood film


of a 23-year-old pregnant woman. Use it to
answer questions 67-69.
Parameter Value Normal

60
ACADEMIC COMMITTEE PBL3B…2025

Platelets 1153/ul 100-300/ul d. Autograft rejection

MPV 5.9 6.5-12.0 72. The following causes of anaemia are due
to membrane defects
PDW 9.0 9.0-17.0
a. HbS
b. Elliptocytosis
c. HbC
67. The most likely haematological d. G6PD deficiency
abnormality in this peripheral blood film
is? 73. Which of the following factors is an
a. Hypochromic normocytic enzyme?
anaemia a. Prothrombin
b. Thrombophilia b. Fibrinogen
c. Spherocytosis c. Thrombin
d. Megaloblastic anaemia d. Factor X

68. This patient may present with the 74. Factor X is activated by the following
following? except?
a. DIC a. VIII
b. Thrombosis b. V
c. Bleeding diathesis c. Calcium
d. Epistasis d. PF3/PL

69. The most appropriate test to request for 75. Factor XII is activated by
this patient is? a. V
a. PTT b. III
b. PT c. II
c. Bleeding time d. VIII
d. Factor assay e. None of the above

70. Blood and or products given to 76. The following coagulation factors are
congestive heart failure due to severe neutralized by serine protease except?
anemia is? a. XII
a. Whole blood b. II
b. Packed red blood cells c. VIII
c. FFP d. IX
d. Platelet concentrate
A woman aged 65 years had palpitations,
easy fatigability, exertional dyspnoea,
71. Hereditary causes of haemolytic anaemia
dizziness and early satiety. The Hb checked
include except?
a. Pyruvate kinase deficiency was 9g/gl, reticulocyte count was 20%. Use
b. HbS it to answer 77-81.
c. HbC
61
ACADEMIC COMMITTEE PBL3B…2025

77. How much blood product should be b. Prostacyclin


given to raise the Hb to 12 g/dl c. NO
a. 450 ml d. Thromboxane A2
b. 900 ml
c. 3 units 84. What acts on fibrinogen to produce
d. 4 units fibrin monomers plus fibrinopeptides A
and B?
78. What type of blood products should be a. Thrombin
given? b. Prothrombin
a. Whole blood c. Factor XIII
b. Platelet concentrate d. Factor III
c. Packed red blood cells
d. Cryoprecipitate 85. Factor VII is activated by
a. III
79. What is the Haematocrit? b. XIII
a. 12% c. VII
b. 27% d. IX
c. 36%
d. 3% 86. The commonest variant of Hodgkin’s
lymphoma?
80. What is the corrected reticulocyte count? a. Lymphocyte depleted
a. 12% b. Lymphocyte predominant
b. 27% c. Nodular sclerosing
c. 36% d. Mixed cellularity
d. 3%
87. The clinical features of extravascular
haemolysis include the following except?
81. What condition accounted for the a. Jaundice
corrected reticulocyte? b. Splenomegaly
a. Aplastic anaemia c. Hepatomegaly
b. Sepsis d. Haemoglobinaemia
c. Adenocarcinoma
d. All of the above 88. The following cross-matching are true
except?
82. Which step of haemostasis is affected in a. Normal bleeding time---4
Glanzmann’s disease? minutes
a. Re-establishment of blood flow b. Abnormal bleeding time—8
b. Dissolution of platelet plug minutes
c. Platelet aggregation agent ---
c. Platelet plug formation
epinephrine
d. Platelet synthesis d. Ristocetin ---evaluate platelet
function
83. The following factors are synthesized by
an intact endothelial cell except?
a. vWF
62
ACADEMIC COMMITTEE PBL3B…2025

a.

63

You might also like